Download as pdf or txt
Download as pdf or txt
You are on page 1of 88

Contents

1 Numbers, Inequalities and Absolute Values 1


1.1 Real Numbers . . . . . . . . . . . . . . . . . . . . . . . . . . . . . . . . . . . . . . . . . . . . . 1
1.2 Inequalities . . . . . . . . . . . . . . . . . . . . . . . . . . . . . . . . . . . . . . . . . . . . . . 2
1.3 Absolute Values . . . . . . . . . . . . . . . . . . . . . . . . . . . . . . . . . . . . . . . . . . . . 4

2 Functions 6
2.1 Functions: Some Basics . . . . . . . . . . . . . . . . . . . . . . . . . . . . . . . . . . . . . . . 6
2.1.1 Even and odd functions . . . . . . . . . . . . . . . . . . . . . . . . . . . . . . . . . . . . 9
2.2 Classification and Combination of Functions . . . . . . . . . . . . . . . . . . . . . . . . . . . . . 10
2.2.1 Classification of functions . . . . . . . . . . . . . . . . . . . . . . . . . . . . . . . . . . 10
2.2.2 Sums, Differences, Products and Quotients . . . . . . . . . . . . . . . . . . . . . . . . . 11
2.2.3 Composite Functions . . . . . . . . . . . . . . . . . . . . . . . . . . . . . . . . . . . . . 12
2.2.4 Shifting, Scaling and Reflecting a Graph . . . . . . . . . . . . . . . . . . . . . . . . . . . 12
2.3 Inverse Functions . . . . . . . . . . . . . . . . . . . . . . . . . . . . . . . . . . . . . . . . . . . 13

3 Angles and Trigonometric Functions 17


3.1 Radian Measure . . . . . . . . . . . . . . . . . . . . . . . . . . . . . . . . . . . . . . . . . . . . 17
3.1.1 Radians Versus Degrees . . . . . . . . . . . . . . . . . . . . . . . . . . . . . . . . . . . 17
3.1.2 Radian measures and arclength . . . . . . . . . . . . . . . . . . . . . . . . . . . . . . . . 18
3.1.3 Area of a Sector . . . . . . . . . . . . . . . . . . . . . . . . . . . . . . . . . . . . . . . 20
3.2 Review of Trigonometric Functions . . . . . . . . . . . . . . . . . . . . . . . . . . . . . . . . . 22
3.2.1 Revision: Defining the Trigonometric Functions . . . . . . . . . . . . . . . . . . . . . . 22
3.2.2 Revision: Special Angles . . . . . . . . . . . . . . . . . . . . . . . . . . . . . . . . . . . 22
3.2.3 Revision: Trigonometric Periods . . . . . . . . . . . . . . . . . . . . . . . . . . . . . . . 23
3.2.4 Revision: Some Important Trigonometric Identities . . . . . . . . . . . . . . . . . . . . . 23
3.3 Inverse Trigonometric Functions . . . . . . . . . . . . . . . . . . . . . . . . . . . . . . . . . . . 26
3.3.1 The arcsine Function . . . . . . . . . . . . . . . . . . . . . . . . . . . . . . . . . . . . . 26
3.3.2 The arccosine Function . . . . . . . . . . . . . . . . . . . . . . . . . . . . . . . . . . . . 29
3.3.3 The arctangent Function . . . . . . . . . . . . . . . . . . . . . . . . . . . . . . . . . . . 30
3.4 Trigonometric Equations . . . . . . . . . . . . . . . . . . . . . . . . . . . . . . . . . . . . . . . 32
3.4.1 General Solutions to Trigonometric Equations . . . . . . . . . . . . . . . . . . . . . . . . 32
3.5 Polar Co-ordinates . . . . . . . . . . . . . . . . . . . . . . . . . . . . . . . . . . . . . . . . . . 34
3.5.1 Cartesian Co-ordinates and Polar Co-ordinates . . . . . . . . . . . . . . . . . . . . . . . 34
3.5.2 Converting Polar co-ordinates to Cartesian Co-ordinates and vice versa . . . . . . . . . . 35

i
ii 1st Semester Algebra Lecture Manual 2017 MATH1034

3.5.3 Polar Equations . . . . . . . . . . . . . . . . . . . . . . . . . . . . . . . . . . . . . . . . 36


3.5.4 Polar Graphs . . . . . . . . . . . . . . . . . . . . . . . . . . . . . . . . . . . . . . . . . 37
3.5.5 Questions for you to investigate: . . . . . . . . . . . . . . . . . . . . . . . . . . . . . . . 43
3.5.6 Summary . . . . . . . . . . . . . . . . . . . . . . . . . . . . . . . . . . . . . . . . . . . 43
3.6 Expressions of the Form a cos x + b sin x . . . . . . . . . . . . . . . . . . . . . . . . . . . . . . . 44

4 Mathematical Induction 46
4.1 Introduction . . . . . . . . . . . . . . . . . . . . . . . . . . . . . . . . . . . . . . . . . . . . . . 46
4.2 The Principle of Mathematical Induction . . . . . . . . . . . . . . . . . . . . . . . . . . . . . . . 47

5 Sigma Notation and Binomial Theorem 52


5.1 Sigma Notation . . . . . . . . . . . . . . . . . . . . . . . . . . . . . . . . . . . . . . . . . . . . 52
5.1.1 Introduction to Sigma Notation . . . . . . . . . . . . . . . . . . . . . . . . . . . . . . . 52
5.1.2 Revision of Formulae for Some Simple Sums . . . . . . . . . . . . . . . . . . . . . . . . 55
5.2 Factorials . . . . . . . . . . . . . . . . . . . . . . . . . . . . . . . . . . . . . . . . . . . . . . . 59
5.3 The Binomial Theorem . . . . . . . . . . . . . . . . . . . . . . . . . . . . . . . . . . . . . . . . 61

6 Conic Sections 66
6.1 Quadratic Forms . . . . . . . . . . . . . . . . . . . . . . . . . . . . . . . . . . . . . . . . . . . 66
6.1.1 Parabola . . . . . . . . . . . . . . . . . . . . . . . . . . . . . . . . . . . . . . . . . . . . 67
6.1.2 Ellipse . . . . . . . . . . . . . . . . . . . . . . . . . . . . . . . . . . . . . . . . . . . . 67
6.1.3 Hyperbola . . . . . . . . . . . . . . . . . . . . . . . . . . . . . . . . . . . . . . . . . . . 68
6.2 Change of Axes . . . . . . . . . . . . . . . . . . . . . . . . . . . . . . . . . . . . . . . . . . . . 70
6.2.1 Translation of axes . . . . . . . . . . . . . . . . . . . . . . . . . . . . . . . . . . . . . . 70
6.2.2 Rotation of axes . . . . . . . . . . . . . . . . . . . . . . . . . . . . . . . . . . . . . . . 75

A Some Basic Mathematical Notions A1


A.1 Basic Mathematical Terms, Sets and Forms of Reasoning . . . . . . . . . . . . . . . . . . . . . . A1
A.1.1 Basic Mathematical Terms . . . . . . . . . . . . . . . . . . . . . . . . . . . . . . . . . . A1
A.1.2 Forms of Reasoning . . . . . . . . . . . . . . . . . . . . . . . . . . . . . . . . . . . . . A2
A.1.3 Methods of proof . . . . . . . . . . . . . . . . . . . . . . . . . . . . . . . . . . . . . . . A3
A.1.4 Sets . . . . . . . . . . . . . . . . . . . . . . . . . . . . . . . . . . . . . . . . . . . . . . A4
A.2 Real Numbers and Intervals . . . . . . . . . . . . . . . . . . . . . . . . . . . . . . . . . . . . . . A5
A.2.1 Definition and Properties of Real Numbers . . . . . . . . . . . . . . . . . . . . . . . . . A5
A.2.2 Intervals . . . . . . . . . . . . . . . . . . . . . . . . . . . . . . . . . . . . . . . . . . . . A8
A.3 The Power of Notation and Reasoning . . . . . . . . . . . . . . . . . . . . . . . . . . . . . . . . A8
Chapter 1

Numbers, Inequalities and Absolute


Values

In this chapter important basic notations and concepts are reviewed. More details are given in the appendix and in
the prescribed textbooks, in particular Thomas, Appendix B.1, and Huntley/Love, Section 1.1.

LEARNING OUTCOMES:
On completion of this chapter you should (tick the checkbox when you have mastered the skill)
 1. know how to describe the integers, the rational numbers and the irrational numbers,
 2. understand that the real numbers consist of the rational numbers and and the irrational numbers,
 3. understand the meaning of and the notation for intervals,
 4. be able to distinguish between open intervals and closed bounded intervals,
 5. understand the meaning of inequalities and be able to write the solutions in set notation,
 6. know and understand the definition of the absolute value,
 7. know and be able to apply the identity |a| = √a , 2

 8. know how to solve the examinable tutorial problems and the worked out problems from these notes on
this chapter.

1.1 Real Numbers


The natural numbers are all numbers 1, 2, 3, . . . . The set of natural numbers is denoted by N. In set notation
this can be written as
N = {1, 2, 3, . . . }.
The integers consist of all natural numbers, negatives of natural numbers, and zero. The set of integers is denoted
by Z. In set notation this can be written as
Z = {. . . , −3, −2, −1, 0, 1, 2, 3, . . . }.
p
The rational numbers consist of all quotients , where p and q are integers with q , 0. The set of rational
q
numbers is denoted by Q. In set notation this can be written as
ß ™
p
Q= : p, q ∈ Z, q , 0 .
q

The real numbers are obtained from the rational numbers by filling in all the “holes” on the number line. The
exact definition will be given in second year in Basic Analysis. The set of real numbers is denoted by R.

1
2 1st Semester Algebra Lecture Manual 2017 MATH1034

The irrational numbers consist of all real numbers which are not rational numbers. There is no particular symbol
for the irrational numbers, but one may use the notations R \ Q or Q0 .
For more details and the algebraic operations of addition, subtraction, multiplication and divison of real numbers,
see Subsection A.2.1 in the Appendix.
Real numbers can be plotted on the real number line:

R

−1 0 1 2 2 3π 4

Real numbers can be ordered: If a and b are two real numbers, then we say that “b is greater than a” or “b is larger
than a” or “a is less than b” or “a is smaller than b” if b is to the right of a on the real number line, and we write

b>a or, equivalently a < b.

It is convenient to introduce the following notation for two real numbers a and b:

a ≤ b ⇔ a < b or a = b,
a ≥ b ⇔ a > b or a = b.

Worked Example 1.1.1. The following statements are all true:

5
1 < 2, 1 ≤ 2, 5 ≥ 5, π ≤ π, 2.5 ≤ .
2

Intervals are subsets of the real number line which correspond geometrically to line segments. For example, if
a < b are real numbers, then the open interval from a to b is the set of all real numbers strictly between a and b
and is denoted by (a, b):
(a, b) = {x ∈ R | a < x < b}.
If we include the endpoint, then we obtain the closed interval from a to b, denoted by [a, b]:

[a, b] = {x ∈ R | a ≤ x ≤ b}.

See Subsection A.2.2 in the Appendix for the definitions and notations of all possible intervals.

Tutorial 1.1.1. 1. If an intersection of intervals contains more than one number, then this intersection is again an
interval. Hence write the following intersections in interval notation:
(a) (−2, 10) ∩ (−10, 2), (b) (−2, 5) ∩ [2, 10), (c) (−∞, π) ∩ (1, 25 8 ), (d) (2, ∞) ∩ (−∞, 9).
2. For each of the following sets decide whether it is an interval. If it is, write it in interval notation. If it is not,
justify your decision:
(a) [−2, 5) ∪ (3, 7), (b) (−∞, 0) ∪ (0, ∞), (c) [−2, 2) ∪ [2, 5], (d) (−∞, 3] ∪ [5, 8).

1.2 Inequalities
In this section you will learn how to write the set of real numbers satisfying one or more inequalities involving
that real number. This is also called solving the inequality.

Worked Example 1.2.1. 1. Find the set of all real numbers x which satisfy the inequalities
a) 3x + 1 > 2x OR 3x + 1 ≤ 5x − 2,
b) 3x + 1 > 2x AND 3x + 1 ≤ 5x − 2.
c) Hence solve 2x < 3x + 1 ≤ 5x − 2.
Give the solutions in both interval and set notation (see Appendix A.2.2).
MATH1034 1st Semester Algebra Lecture Manual 2017 3

Solution. a)

3x + 1 > 2x ⇔ x + 1 > 0 subtract 2x on both sides


⇔ x > −1 subtract 1 on both sides

and hence
{x ∈ R | 3x + 1 > 2x} = {x ∈ R | x > −1} = (−1, ∞),
and

3x + 1 ≤ 5x − 2 ⇔ 1 ≤ 2x − 2 subtract 3x on both sides


⇔ 3 ≤ 2x add 2 on both sides
3
⇔ ≤ x divide both sides by 2
2
3
⇔x≥ interchange sides
2
and hence ß ™ ï ã
3 3
{x ∈ R | 3x + 1 ≤ 5x − 2} = x∈R|x≥ = ,∞ .
2 2
Therefore
ß ™
3
{x ∈ R | 3x + 1 > 2x or 3x + 1 ≤ 5x − 2} = x ∈ R | x > −1 or x ≥ = {x ∈ R | x > −1} = (−1, ∞).
2
b)
ß ™ ß ™ ï ã
3 1 3
{x ∈ R : 3x + 1 > 2x and 3x + 1 ≤ 5x − 2} = x ∈ R | x > −1 and x ≥ = x∈R|x≥3 = ,∞ .
2 2 2

c) Since 2x < 3x + 1 ≤ 5x − 2 means 3x + 1 > 2x and 3x + 1 ≤ 5x − 2, the problem and therefore the solution is
that of part (b).
4 3
2. Find all real numbers x which satisfy + ≤ 1.
x x−2
Solution. First we observe that the left hand side is not defined if x = 2 or x = 0. Hence we assume x , 0 and
x , 2. Then
4 3 4 3
+ ≤1⇔ + − 1 ≤ 0 (subtract 1 on both sides)
x x−2 x x−2
4(x − 2) + 3x − x(x − 2)
⇔ ≤ 0 (take the common denominator)
x(x − 2)
4x − 8 + 3x − x2 + 2x
⇔ ≤0
x(x − 2)
−x2 + 9x − 8
⇔ ≤ 0 (collect terms)
x(x − 2)
x2 − 9x + 8
⇔ ≥0
x(x − 2)
(x − 1)(x − 8)
⇔ ≥ 0.
x(x − 2)

The expression changes its sign when one of the factors in the numerator or denominator is zero, which occurs
when x has the values 0, 1, 2, or 8. Since all factors are positive when x > 8, we have the following sign table:

+ − + − +
x
0 1 2 8
4 1st Semester Algebra Lecture Manual 2017 MATH1034

Hence, the solution set is

{x ∈ R : x < 0 or 1 ≤ x < 2 or x ≥ 8} = (−∞, 0) ∪ [1, 2) ∪ [8, ∞).

Tutorial 1.2.1. 1. 1. Huntley/Love Exercises 3.2A p. 155 (earlier edition p. 130): 19, 21, 28, 54,55, 13, 14, 17,
23.
2. Solve the inequalities
1 2
(a) ≤7 (b) x + ≥ 4.
x+4 x−1
3. Let a, b ∈ [0, ∞) with a2 ≤ b2 . Show that a ≤ b.

1.3 Absolute Values


The absolute value of a real number a is defined by
(
a if a ≥ 0,
|a| =
−a if a < 0.

Note. 1. If a and b are real numbers with b ≥ a, then b − a ≥ 0 is the distance between these two numbers on the
real number line. Hence (
a − 0 if a ≥ 0,
|a| =
0 − a if a < 0.
is the distance from a to 0.
2. Either from part 1. or from the fact that −a > 0 if a < 0 it follows that |a| ≥ 0 for all a and that |a| = 0 if and
only if a = 0.

√ without proof, that for each real number r ≥ 0 there is exactly one number x ≥ 0 with x = r.
2
Below we will use,
x is denoted by r and is called the square root of r.

Theorem 1.1. For each real number a we have


1. |a| ≥ a,
2. |a|2 = a2 ,

3. |a| = a2 .

Proof. 1. If a > 0, then


|a| = a ≥ a,
and if a < 0, then −a > 0 and therefore
|a| = −a ≥ 0 ≥ a.
2. If a ≥ 0, then |a| = a gives
|a|2 = a2 ,
whereas for a < 0, |a| = −a gives

|a|2 = (−a)2 = ((−1)a)2 = (−1)2 a2 = (1)a2 = a2 .

3. follows now from 2. and |a| ≥ 0. 

Theorem 1.2 (Absolute Value Properties). Let a and b be real numbers. Then
1. | − a| = |a|,
2. |ab| = |a| |b|,
a |a|
3. = if b , 0,
b |b|
MATH1034 1st Semester Algebra Lecture Manual 2017 5

4. |a + b| ≤ |a| + |b| (triangle inequality),


5. |a| < b ⇔ −b < a < b if b ≥ 0,
6. |a| > b ⇔ a < −b or b < a, if b ≥ 0.

Proof. The proof of properties 1, 2, 3, 5 is a tutorial problem.


4. Using Theorem 1.1, we find

|a + b|2 = (a + b)2 Theorem 1.1.2


= a + 2ab + b2
2

≤ |a|2 + 2|a||b| + |b|2 part 2 and Theorem 1.1.1,2


= (|a| + |b|)2
Theorem 1.1.2

Then Tutorial 1.2.1.3 gives |a + b| ≤ |a| + |b|.


6. If a ≥ 0, then
|a| > b ⇔ a > b,
and if a < 0, then
|a| > b ⇔ −a > b ⇔ a < −b. 

Worked Example 1.3.1. Solve the inequalities


a) |2x + 4| < 1, b) 3 ≤ |5x − 6|.
Solution. a)

|2x + 4| < 1 ⇔ −1 < 2x + 4 < 1


⇔ −5 < 2x < −3
5 3
⇔− <x<− .
2 2
b)

3 ≤ |5x − 6| ⇔ |5x − 6| ≥ 3
⇔ 5x − 6 ≥ 3 or 5x − 6 ≤ −3
⇔ 5x ≥ 9 or 5x ≤ 3
9 3
⇔ x ≥ or x ≤ .
5 5
Tutorial 1.3.1. 1. Huntley/Love Exercises 3.4A p. 172 (earlier edition p. 147): 31, 33, 37, 44, 48.
2. Huntley/Love Exercises 3.4C p. 149: 67, 90, 92, 93.
3. a) Prove properties 1, 2, 3, 5 of Theorem 1.2.
b) Prove the triangle inequality directly, using the definition of the absolute value.
4. Let r ≥ 0. Find all real numbers a such that |a| = r. How many numbers did you find?
Chapter 2

Functions

Thomas, Chapter 1 relates to this chapter.

LEARNING OUTCOMES:
On completion of this chapter you should (tick the checkbox when you have mastered the skill)

 1. understand the notion of a function, and be able to find the domain and the range of a function,
 2. know and be able to identify the graph of a function,
 3. be able to determine whether a function is even or odd,
 4. be able to identify polynomials, rational functions and algebraic functions,
 5. be able to find sums, differences, products, quotients and composites of functions,
 6. be able to sketch the graphs of shifted, scaled and reflected functions,
 7. know and understand the definition of one-to-one functions and be able to decide whether a function is
one-to-one,
 8. know and understand the definition of the inverse of a function, be able to find the inverse, and know how
to sketch its graph,
 9. know how to solve the examinable tutorial problems and the worked out problems from these notes on
this chapter.

2.1 Functions: Some Basics


Definition: Function

A function f from a set D to a set Y is a rule that assigns a unique (single) element f (x) ∈ Y to each
element x ∈ D.

The element f (x) is called the image of x under f .


The set D is called the domain of f and denoted by dom( f ).
The set { f (x) | x ∈ D} is called the range of f and denoted by range( f ). Note that the range is a subset of Y,
consisting of the images under f of all elements in D.
In particular when D ⊂ R and Y = R one often writes y = f (x) and calls x the independent variable and y the
dependent variable. Such functions are also called real-valued.

6
MATH1034 1st Semester Algebra Lecture Manual 2017 7

A symbolic sketch of a function may look like this:

D Y
Domain
Range
x

f (x)

Note. 1. We have to distinguish between the function f and the image f (x) of a particular x ∈ D. However, it is
sometimes convenient to keep x as a dummy variable when one refers to the function. For example, one would
rather write “the function x3 + x − 1 is a polynomial” than “if the function f is defined by f (x) = x3 + x − 1, then
f is a polynomial”.
2. The symbol which we use to name the function or the variable does not affect the values of the function. For
example, the functions f and g, given by f (x) = x3 + x − 1 and g(t) = t3 + t − 1 with domain R, define the same
function because dom( f ) = dom(g) and f (x) = g(t) for all values of x = t in the common domain of f and g. For
this reason, variables are also referred to as “dummy variables”.
3. In particular for functions f where we substitute real values x for f (x) we call those values of x for which f (x)
gives a well-defined real number the natural domain of f .

Worked Example 2.1.1. Let


1 1
a) f (x) = √ , b) g(x) = .
x−1 |x| − 2
Find the natural domain for each of these functions.
Solution. a) The square root is√defined for non-negative real numbers, and so x−1 ≥ 0, i. e., x ≥ 1 is needed. Since
we cannot divide by zero and 0 = 0, we have to exclude x = 1. Hence the natural domain of f is dom f = (1, ∞).
b) The absolute value is defined for all x ∈ R, but we cannot divide by zero, so |x| − 2 , 0 is required. Hence the
natural domain is
dom g = {x ∈ R | x , 2 and x , −2} = (−∞, −2) ∪ (−2, 2) ∪ (2, ∞).

Worked Example 2.1.2. a) Let f (x) = −2x + 1 with domain [0, 1).
1
b) Let g(x) = 2 with natural domain.
x +1
Find the range of each of the functions f and g.
Solution. a)
x ∈ [0, 1) ⇔ 0 ≤ x < 1
⇔ 0 ≥ −2x > −2
⇔ 1 ≥ −2x + 1 > −1
⇔ −1 < −2x + 1 ≤ 1.
Hence range( f ) = (−1, 1].
1 1
b) Clearly 1 ≤ x2 + 1, so that y = satisfies 0 < y ≤ 1. On the other hand, for y ∈ (0, 1] we have ≥ 1, and
+1 x2 y
1 1
the equation y = 2 has a solution x = − 1 (this solution is not necessarily unique):
x +1 y

1 1 1 1
x= − 1 ⇒ x2 = − 1 ⇒ x2 + 1 = ⇒ y = 2 ,
y y y x +1
so that each y ∈ (0, 1] belongs to the range of g. Hence range(g) = (0, 1].
8 1st Semester Algebra Lecture Manual 2017 MATH1034

Definition: Graph of a Function

Let f : D → Y. We define the graph of f to be the set {(x, f (x)) | x ∈ D}.

If D ⊂ R and Y = R (or Y ⊂ R) the graph is a subset of the Cartesian plane R2 .

Example 2.1.3. The functions in Worked Examples 2.1.1 and 2.1.2 have the following graphs:

y
y
y
y
1
1 1
1
x
x
−2 2
x 1 x
1 1

1 1 y = −2x + 1 1
y= √ y= y=
x−1 |x| − 2 (x ∈ [0, 1)) x2 + 1

Therefore, graphs of functions from D ⊂ R to R can be represented by curves in the Euclidean plane. However,
not every curve in the Euclidean plane represents a function.

Worked Example 2.1.4. The graph of the unit circle, i, e., the set of points in the plane satisfying x2 + y2 = 1 is
not the graph of a function.
Solution.

y y y

x x x
−1 1 −1 1 −1 1

√ √
x2 + y2 = 1 y= 1 − x2 y = − 1 − x2

We see that for every x ∈ (−1, 1) there are two y-values such that (x, y) lies on the circle:
√ √
y = 1 − x2 and y = − 1 − x2 .
However, the upper and lower semicircle define functions.

Vertical Line Test

A curve in the plane is the graph of a function if and only if each vertical line intersects the
curve at most once.
That is, if (x, y1 ) and (x, y2 ) belong to the curve, then y1 = y2 .

Numerical Representation of a curve


Another way of representing a curve is as a table of pairs of x-values and y-values. However, in this way only
finitely many points of the graph are represented, and if the function is to be defined for infinitely many values,
MATH1034 1st Semester Algebra Lecture Manual 2017 9

then a rule for “interpolating” and “extrapolating” these values must be given.

Note. 1. In Algebra and Calculus we will only be concerned with exactly defined functions. Hence the graph of a
function serves mostly as an illustration or a tool to derive qualitative properties of the function. You will see the
latter for example in the Calculus chapter on Applications of Differentiation.
2. Graphical and numerical representations of curves may determine the curve exactly if one specifies particular
curves like parabolas, straight lines, circles, etc., see Examples 2.1.2 a), 2.1.3, 2.1.4.
A function may be piecewise-defined, that is, it may be given by different formulas on different parts of its domain.

Example 2.1.5. 

 x3 if x ≤ 0,

f (x) = 2x if 0 < x < 2,


 2
x if x ≥ 2.

A function f is called increasing if its graph rises, that is, algebraically, if f (x1 ) < f (x2 ) whenever x1 < x2 .
A function f is called decreasing if its graph falls, that is, algebraically, if f (x1 ) > f (x2 ) whenever x1 < x2 .

Tutorial 2.1.1. 1. Find the natural domain and range of (a) f (x) = 1 + x2 ; (b) f (x) = 1 − x;
√ √ 4 2
(c) f (x) = 3x + 10; (d) g(x) = x2 − 3x; (e) f (t) = ; (f) G(t) = 2 .
3−t t − 16
2. Huntley/Love Exercises 2.3A p. 259 (earlier edition p. 77): 20, 22, 24, 25, 27.
3. Show that the function in Example 2.1.5 is increasing, either algebraically or by sketching its graph.

2.1.1 Even and odd functions


In this subsection, all functions are real-valued functions defined on a symmetric subset D of R, where D is called
symmetric if x ∈ D ⇒ −x ∈ D. For example,

(−2, 2) and [−2, 2] are symmetric, whereas (−2, 2] and (−2, 3) are not symmetric.

A function f is called an even function if f (−x) = f (x) for all x ∈ D.

Note. The function f is even if and only if the graph of f is symmetric about the y-axis, i.e., (x, y) belongs to the
graph of f if and only if (−x, y) belongs to the graph f .

Example 2.1.6. The functions f (x) = 21 x2 − 1 and g(x) = |x| are even.

y y

f g

A function f is called an odd function if f (−x) = − f (x) for all x ∈ D.

Note. The function f is odd if and only if the graph of f is symmetric about the origin, i.e., (x, y) belongs to the
graph of f if and only if (−x, −y) belongs to the graph f .

Example 2.1.7. The functions f (x) = 21 x and g(x) = 12 x3 − x are odd.


10 1st Semester Algebra Lecture Manual 2017 MATH1034

y
g
y
f

x x

Worked Example 2.1.8. Show that


a) f (x) = x2 + 2x4 + 1 is an even function;
Solution. The domain of f is R, which is symmetric, and
f (−x) = (−x)2 + 2(−x)4 + 1 = (−1)2 x2 + 2(−1)4 x4 + 1 = x2 + 2x4 + 1 = f (x)
shows that f is an even function.
b) f (x) = x − x3 is an odd function;
Solution. The domain of f is R, which is symmetric, and
f (−x) = (−x) − (−x)3 = −x − (−1)3 x3 = −x + x3 = −(x − x3 ) = − f (x)
shows that f is an odd function.
c) f (x) = 3x4 + 2x is neither even nor odd.
Solution. The domain of f is R, which is symmetric, and
f (1) = 3(1)4 + 2(1) = 5, f (−1) = 3(−1)4 + 2(−1) = 1,
So neither f (−1) = f (1) nor f (−1) = − f (1). Hence f is neither even nor odd.

2.2 Classification and Combination of Functions


2.2.1 Classification of functions
A function p is called a polynomial if it is of the form
p(x) = an xn + an−1 xn−1 + · · · + a2 x2 + a1 x + a0 ,
where n is a nonnegative integer and the numbers an , an−1 , . . . , a2 , a1 , a0 are real numbers with an , 0. The number
n is called the degree of the polynomial p, and the numbers an , an−1 , . . . , a2 , a1 , a0 are called the coefficients of
the polynomial p. The natural domain of a polynomial is R.
If n = 0, then the polynomial p is called a constant function.
If n = 1, then the polynomial p is called a linear function.

Example 2.2.1. 5x4 + 3x2 − 4x + 1 is a polynomial.

A function g is called a rational function if it has the form


p(x)
g(x) = ,
q(x)
where p, q are polynomials and q is not identically 0. Then the natural domain of the rational function g is
dom(g) = {x ∈ R | q(x) , 0}.
MATH1034 1st Semester Algebra Lecture Manual 2017 11

2x6 + 4x3 − x2 + 8
Example 2.2.2. is a rational function.
5x7 − 3x2

Polynomials and rational functions are examples of algebraic functions. A function is called algebraic if it can be
constructed using algebraic operations (addition, subtraction, multiplication, division and powers or roots).
√ x3 − 5
Example 2.2.3. x5 − 2 and √3 + 4x3 + 1 are algebraic functions.
x2 − 1

You will be presented with further classes of functions as this course proceeds.
√7
Tutorial 2.2.1. 1. Classify the following functions: (a) f (x) = 5 − 6x; (b) f (x) = x2 − 7;
x2 − 2
(c) f (x) = 2 .
x +2
2. Huntley/Love Exercises 2.3A p. 259 (earlier edition p. 77): 33, 38, 42.

2.2.2 Sums, Differences, Products and Quotients


f
We can combine two functions f and g to form new functions f + g, f − g, f g, using the following definitions:
g

Function Definition domain


f +g ( f + g)(x) = f (x) + g(x) dom( f + g) = dom( f ) ∩ dom(g)
f −g ( f − g)(x) = f (x) − g(x) dom( f − g) = dom( f ) ∩ dom(g)
fg ( f g)(x) = f (x)g(x) dom( f g) = dom( f ) ∩ dom(g)
Å ã Å ã
f f f (x) f
(x) = dom = dom f ∩ {x ∈ dom g | g(x) , 0}
g g g(x) g

Worked Example 2.2.4. Let f (x) = x and g(x) = x2 − 4. Find the following functions and determine their
f
domains: f + g, f − g, f g, .
g
Solution. From dom f = [0, ∞) and dom g = R we find
dom( f + g) = dom( f − g) = dom( f g) = dom( f ) ∩ dom(g) = [0, ∞).
Then

( f + g)(x) = f (x) + g(x) = x + x2 − 4,

( f − g)(x) = f (x) − g(x) = x − x2 + 4,

( f g)(x) = f (x)g(x) = x(x2 − 4).
Finally, g(x) = 0 ⇔ x = 2 or x = −2 gives
Å ã
f
dom = {x ∈ [0, ∞) | x , 2} = [0, 2) ∪ (2, ∞)
g
and Å ã √
f x
(x) = 2 .
g x −4
√ 1 f
Tutorial 2.2.2. 1. For (a) f (x) = x2 , g(x) = x − 1; (b) f (x) = x2 − 1, g(x) = √ find f + g, f − g, f g, ,
x +1
2 g
g
. Also find the domains and ranges of these functions.
f
f
2. Let f and g be even or odd. What can be said about f + g, f − g, f · g, ?
g
12 1st Semester Algebra Lecture Manual 2017 MATH1034

2.2.3 Composite Functions

Let f and g be functions. The function f ◦ g defined by ( f ◦ g)(x) = f (g(x)) is called the
composite of f with g.
The domain of f ◦ g is the set of x in the domain of g for which g(x) is in the domain of f ,
i. e., dom( f ◦ g) = {x ∈ dom(g) | g(x) ∈ dom f }.

Note. In general, f ◦ g is not the same as g ◦ f .



Worked Example 2.2.5. Let f (x) = x and g(x) = x2 − 4. Find f ◦ g and g ◦ f .
Solution.

dom( f ◦ g) = {x ∈ R | x2 − 4 ≥ 0}
= (−∞, −2] ∪ [2, ∞),
( f ◦ g)(x) = f (g(x))
= f (x2 − 4)

= x2 − 4,
dom(g ◦ f ) = [0, ∞),
(g ◦ f )(x) = g( f (x))

= g( x)

= x2 − 4
= x−4 (note that x ≥ 0).

Note that we could write (g ◦ f )(x) = x − 4 or (g ◦ f )(x) = |x| − 4.

Worked Example 2.2.6. Show that the composite of two odd functions is odd.
Proof. The domain of f ◦ g is symmetric:

x ∈ dom( f ◦ g) ⇔ x ∈ dom(g) and g(x) ∈ dom( f )


⇔ −x ∈ dom(g) and − g(x) ∈ dom( f ) ∵ dom(g) and dom( f ) are symmetric
⇔ −x ∈ dom(g) and g(−x) ∈ dom( f ) ∵ g is odd
⇔ −x ∈ dom( f ◦ g).

f is odd:

( f ◦ g)(−x) = f (g(−x))
= f (−g(x)) ∵ g is odd
= − f (g(x)) ∵ f is odd
= −( f ◦ g)(x).

Tutorial 2.2.3. 1. For the functions in Tutorial 2.2.2.1, find f ◦ g and g ◦ f ; also find their domain and range.

2. Let f (x) = 1 − x2 , g(x) = 2 + x2 . Find the domain of f ◦ g.
2. Let a) f and g be even, b) f be even and g be odd, c) f be odd and g be even. In each of the cases a), b), c) find
if f ◦ g is even or odd.

2.2.4 Shifting, Scaling and Reflecting a Graph


If f is a real-valued function with domain in R with graph given by y = f (x), then the transformations

• y = f (x + c) and y = f (x) + c for c ∈ R, c , 0, translate the graph,


MATH1034 1st Semester Algebra Lecture Manual 2017 13

• y = f (cx) and y = c f (x) for c ∈ (0, ∞), c , 1, scale the graph,


• y = f (−x) and y = − f (x) reflect the graph.

Tutorial 2.2.4. 1. Graph each function by applying a suitable transformation to reduce the task to sketching a
√ 1
standard function: (a) f (x) = − x + 2; (b) f (x) = (x − 2)3 + 1; (c) f (x) = − 1;
3x
2
(d) f (x) = 2 + 1.
x

2.3 Inverse Functions


Definition: one-to-one

A function f is said to be one-to-one if and only if f (x1 ) = f (x2 ) ⇒ x1 = x2 for all x1 , x2 ∈ dom( f ).

Note. 1. A function f is one-to-one if and only if each value in the range of f is the image of precisely one
element in the domain of f .
2. A function which is one-to-one is also called an injective function.

Horizontal Line Test

A function f is a one-to-one function if and only if each horizontal line intersects its graph at most once.

Example 2.3.1. Apply the vertical and horizontal line tests to decide which of the curves below are graphs of
one-to-one functions.
y y

x x

a) b)
y

x x

c) d)
14 1st Semester Algebra Lecture Manual 2017 MATH1034

The curve in a) is the graph of a function (it satisfies the vertical line test), but fails the horizontal line test. Hence
the function is not one-to-one.
The curve in b) is the graph of a function (it satisfies the vertical line test), and satisfies the horizontal line test.
Hence the function is one-to-one.
The curves in c) and d) fail the vertical line test, and are therefore not graphs of functions.

Note. For a curve to be the graph of a one-to-one function, it must satisfy both the vertical line test and the
horizontal line test.

Worked Example 2.3.2. Let f (x) = x3 . Prove that f is one-to-one.


Proof. Let x1 , x2 ∈ R = dom( f ) such that f (x1 ) = f (x2 ). Then

f (x1 ) = f (x2 ) ⇔ x13 = x23


⇔ x1 = x2 ∵ cubic root is unique

Hence f is one-to-one.

Worked Example 2.3.3. Let f (x) = x2 . Prove that f is not one-to-one.


Proof. Let x1 = 1 and x2 = −1. Then

f (x1 ) = (1)2 = 1 = (−1)2 = f (x2 ).

Hence f (x1 ) = f (x2 ) does not imply x1 = x2 , and so f is not one-to-one.

If f is one-to-one, then for every y ∈ range( f ) there is a unique x ∈ dom( f ) such that y = f (x). Defining

f −1 (y) = x,

we have a new function f −1 with dom( f −1 ) = range( f ) and range( f −1 ) = dom( f ).


The function f −1 is called the inverse of f .

Note. 1. If f is one-to-one and y = f (x), then x = f −1 (y) and therefore

( f ◦ f −1 )(y) = f ( f −1 (y)) = f (x) = y for all y ∈ dom f −1 ,


( f −1 ◦ f )(x) = f −1 ( f (x)) = f −1 (y) = x for all x ∈ dom f −1 .

Note that f ◦ f −1 and f −1 ◦ f may be different functions since their domains are different, in general.
2. For a real number,
1
a−1 =
a
by definition. However, in general, the funtions

1
f −1 and are different
f

since
1
f −1 (x) and are different.
f (x)
Therefore, be careful with your notation. In general, f −1 (x) and ( f (x))−1 are different.
3. One may be able to prove that a function f is one-to-one and find its inverse simultaneously. When solving
y = f (x) for x always gives a unique solution, then (and only then) f is one-to-one, and the obtained values
x = g(y) give the inverse, i. e., f −1 = g.
MATH1034 1st Semester Algebra Lecture Manual 2017 15

x+2
Worked Example 2.3.4. Consider f (x) = 3x + 5 and g(x) = with their natural domains. Show that f and
2x + 1
g are one-to-one and find their inverses.
Solution. a)

y = f (x) ⇔ y = 3x + 5
⇔ y − 5 = 3x
y−5
⇔x= .
3
Since there is a unique solution for all y ∈ R, we have that f is one-to-one, dom f −1 = R, and
5−x
f −1 (x) = .
3
b)
x+2
y = g(x) ⇔ y =
2x + 1
⇔ 2xy + y = x + 2
⇔ 2xy − x = 2 − y (collect terms with x)
⇔ x(2y − 1) = 2 − y
2−y
⇔x= .
2y − 1

1 1
Since there is a unique solution for all y ∈ R with y , and no solution for y = , we have that g is one-to-one,
ß ™ 2 2
1
dom g = x ∈ R | x ,
−1
, and
2
2−x
g−1 (x) = .
2x − 1

Graphs of Inverse Functions


If f is a one-to-one function, then

(x, y) is a point on the graph of f ⇔ y = f (x)


⇔ x = f −1 (y)
⇔ (y, x) is a point on the graph of f −1 .

Therefore, the graph of f −1 is obtained from the graph of f by interchanging the x and y co-ordinates. Geometri-
cally, this corresponds to a reflection across the line y = x:
x x

f f −1

f −1 f

x x

In the next chapter, you will learn about inverses of trigonometric functions. You will also learn more about
inverses in the Calculus module.
16 1st Semester Algebra Lecture Manual 2017 MATH1034

Tutorial 2.3.1. 1. For each of the functions below, find the inverse f −1 and identify the domain and range of f −1 .
1
(a) f (x) = x3 − 2; (b) f (x) = x4 , x ≥ 0; (c) f (x) = 2 , x > 0.
x
2. Let domain f = [−1, 2], domain g = [0, ∞), domain h = (1, ∞) and f (x) = x2 , g(x) = x2 , h(x) = x2 .
Chapter 3

Angles and Trigonometric Functions

LEARNING OUTCOMES:
On completion of this chapter you should (tick the checkbox when you have mastered the skill)

 1. understand the notion of radian measure and be able to convert from degrees to radian measures and vice
versa,
 2. know and understand the connection between arclength and radians,
 3. be able to find the area of a sector,
 4. know and understand the definitions of the trigonometric functions sin, cos, tan, cosec, sec, cot,
 5. know and understand the periods of the trigonometric functions and important trigonometric identities,
 6. know and understand the definitions of the inverse trigonometric functions,
 7. be able to solve trigonometric equations,
 8. know and understand the definition of polar co-ordinates,
 9. be able to convert polar co-ordinates to Cartesian co-ordinates and vice versa,
 10. be able to plot polar graphs in the Cartesian plane and be able to detect symmetry of polar graphs,
 11. be able to write a cos x + b sin x in the form R cos(x − θ),
 12. know how to solve the examinable tutorial problems and the worked out problems from these notes on
this chapter.

3.1 Radian Measure


3.1.1 Radians Versus Degrees
Many ancient civilizations were depending on an accurate calendar for their very existence. The location of the
stars in the night sky is an excellent tool for the identification of the day of the year, and it has been observed that
the stars in the sky move by about 1/360 th of a full circle each night. Hence it was considered to be convenient
to divide a full circle into 360 parts, called degrees in English. The first documented use of this subdivision is by
the ancient Babylonians. Surely they were aware that 360 is not the correct astronomical value, but it is such a
convenient number that it has been in use ever since. Indeed, 360 is the smallest natural number which is divisible
by all natural number from 2 to 10 except 7, and 360 is also the smallest natural number which is divisible by all
natural number from 2 to 12 except 7 and 11. Therefore, all frequently occuring special angles in geometry can
be expressed as integers if one uses degrees.
However, degrees have two drawbacks. Firstly, although convenient in geometry, the divison of the circle into 360
parts is quite arbitrary. And secondly, the use of degrees would introduce some factors which would make many
formulas very clumsy, in particular in calculus.

17
18 1st Semester Algebra Lecture Manual 2017 MATH1034

On the other hand, radians, to be defined right now, can be defined unambiguously by geometric measurements.
For this consider a sector with angle α◦ (measured in degrees because we want to compare degrees with radians
later; stricly speaking, a unit of measurement is only needed when a numerical value is given to the angle):

s
r

For fixed radius r the arclength s is proportional to the angle of the sector (for example, if we double the angle, we
double the arclength; if we halve the angle, we halve the arclength; etc.). The circumferemce of the circle (i. e.,
the arclength of a sector with angle 360◦ ) is proportional to the radius r (and thus the diameter d = 2r) of the
circle. This proportionality factor with respect to the diameter is denoted by π, and therefore

(circumference of circle with radius r) = πd = 2πr.

Recall that to five decimal places, π ≈ 3.14159.


So if an angle of α◦ gives an arclength of s, then

s α◦
= (by proportionality).
2πr 360◦
Solving for s gives
α◦
s = 2πr ,
360◦
i. e.,
π
s= rα◦ (3.1.1)
180◦

3.1.2 Radian measures and arclength


By (3.1.1), for any circle, the angle α (in degrees) is proportial to the ratio arclength s over radius r, and we take
this ratio as definition of the radian measure:

s
α= is called the radian measure of the angle α.
r

Therefore, the formula for arclength becomes very easy if you use the radian measure for the angle:

s = αr where s is arclength, r is radius and α is the angle measured in radians.

From (3.1.1) we obtain the following rules to convert from radians to degrees or vice versa:

Radians to degrees Degrees to radians


180 π
multiply by multiply by
π 180
MATH1034 1st Semester Algebra Lecture Manual 2017 19

Worked Example 3.1.1. Convert the following angles from degrees into radians:
30◦
30◦ , 50◦ , 510◦ , .
π
Solution.
π π
30◦ = 30 · = ,
180 6
π 5π
50◦ = 50 · = ,
180 18
π 17π
510◦ = 510 · = ,
180 6
30◦ 30◦ π 1
= · = .
π π 180 6
Worked Example 3.1.2. Convert the following angles given in radians into degrees:
π π
1, 3, , .
4 7
Solution.
180 ◦ 180 ◦
Å ã Å ã
1= 1· =
π π
180 ◦ 540 ◦
Å ã Å ã
3= 3· = ,
π π
π π 180
Å ã◦
= · = 45◦ ,
4 4 π
π π 180 ◦ 180 ◦
Å ã Å ã
= · = .
7 7 π 7
Worked Example 3.1.3. Find the arclength of a sector subtended by angle 2 radians and radius 6 cm.
Solution. The arclength is
s = rα = (6 cm) · 2 = 12 cm.

Note. 1. No symbol is attached to the angle in radian measure. Radians are quotients of lengths and therefore
dimensionless quantities.
2. From now on, angles will be given in radians, unless stated otherwise.
3. We have the following sign convention:
anti-clockwise angles are signed positive;
clockwise angles are signed negative.
It is very useful to remember the most commonly used angles in radian measure, in particular those given in
Example 3.1.4 below.

Worked Example 3.1.4. Convert the following angles into radians:


360◦ , 180◦ , 90◦ , 60◦ , 45◦ , 90◦ .
Solution.
360◦ = 2π,
π
180◦ = 180 · = π,
180
π π
90◦ = 90 · = ,
180 2
π π
45 = 45 ·

= ,
180 4
π
30◦ = .
6
20 1st Semester Algebra Lecture Manual 2017 MATH1034

Worked Example 3.1.5. Sketch a sector which has its arclength s equal to its radius r and sketch an equilateral
triangle of side length r. Compare the angles.
Solution.

We note that the angle α subtended by the arc is slightly smaller than the angle of 60◦ of the equilateral triangle.
Indeed,
180 ◦
Å ã
α=
π
is about 5% smaller than 60◦ since 3 is approximately 5% smaller than π.

Tutorial 3.1.1. 1. Convert the following angles measured in degrees to angles measured in radians:
(a) 120◦ (b) 150◦ (c) 40◦ (d) 225◦ (e) 330◦
2. Convert the following angles measured in radians to angles measured in degrees:
3π 7π 2π 3
(a) 3π (b) (c) (d) (e)
8 3 5 8
3. Convert the angular speed of 6 revolutions per second to radians per minute.
4. A car is moving at a rate of 80 km/h. The diameter of its wheels is 60 cm.
(a) Find the number of revolutions per minute that the wheels are rotating.
(b) Find the angular speed of the wheels in radians per second.
5. The pointer on a bathroom scale is 3 cm long. Find the angle through which it rotates when its tip moves 4cm
along the scale.

3.1.3 Area of a Sector


The area of a sector with radius r and subtended by an angle α is proportional to the angle α if the radius is fixed.
The area of the full circle of radius r is πr2 .
So if an angle of α gives an area of A, then
A α
= (by proportionality).
πr 2 2π
Solving for A we obtain:

1 2
The area A of a sector of radius r subtended by an angle α is A = r α.
2

Worked Example 3.1.6. Find the arclength and the area of the sector with angle 40◦ and radius 2.
Solution. In radians, the angle is
π 2π
α = 40 · = .
180 9
Therefore, the arclength is
2π 4π
s = rα = (2) = ,
9 9
MATH1034 1st Semester Algebra Lecture Manual 2017 21

and the area is


1 2 1 2π 4π
A= r α = (22 ) = .
2 2 9 9

Worked Example 3.1.7. The total perimeter of a sector of a circle is 4. Find an expression for its area in terms of
(i) its radius r,
(ii) its subtended angle α.
Solution. The perimeter of a sector is formed by two radii and the arc. So the total perimeter of the sector is

2r + s = 2r + rα = (2 + α)r,

and therefore
4 = (2 + α)r (∵ total perimeter is 4).

(i) Since the area A is to be expressed in terms of r, express α in terms of r, i. e., solve the above equation for α:

4 4
2+α= ⇒ α = − 2.
r r

Then the area is


Å ã
1 2 1 4
A= r α = r2 − 2 = 2r − r2 .
2 2 r
(ii) Here we solve the perimeter equation for r:

4
r= .
2+α

Then the area is


Å ã2
1 1 4 8α
A = r2 α = α= .
2 2 2+α (2 + α)2
Tutorial 3.1.2. 1. Find the arclength and area of the sector of a circle with

(a) radius 3, subtending an angle of ;
5
4 7π
(b) radius , subtending an angle of ;
7 3
2. A patio is in the shape of a sector, radius 3 m and subtending angle of 60◦ is to be tiled. Two equally competent
tilers are approached. Tiler A charges R 100 per square metre. Tiler B charges R 50 per metre around the perimeter
of the sector. Which tiler offers the better value for money?
3. A Chinese fan made of silk and fully edged with lace is made in the shape of a sector with angle 115◦ and
radius 20 cm. What area of silk is required and what length of lace is required?
22 1st Semester Algebra Lecture Manual 2017 MATH1034

3.2 Review of Trigonometric Functions


3.2.1 Revision: Defining the Trigonometric Functions
In the diagram y
(x, y)

θ
x

(x, y) is a point on the circumference of the circle with centre (0, 0) and radius r.
The following six trigonometric functions are defined:
y r
sin θ = cosec θ =
r y

x r
cos θ = sec θ =
r x
y x
tan θ = cot θ =
x y
Note. The trigonometric functions sin and cos have values for all angles θ. The remaining four trigonometric
functions are undefined at some values of θ.
The graphs of the trigonometric functions are given in Thomas, p. AP-23.

3.2.2 Revision: Special Angles


For some particular angles, trigonometric functions have values which are rational numbers or which can be
written in surd form. These values either can be read off the above diagram or can be calculated with the aid of
formulas given in the following subsections.

0◦ 30◦ 45◦ 60◦ 90◦


θ π π π π
0
6 4 3 2

1 1 3
sin θ 0 √ 1
2 2 2

3 1 1
cos θ 1 √ 0
2 2 2
1 √
tan θ 0 √ 1 3 undefined
3

Note. If θ is not a special angle, leave its value under a trigonometric function unevaluated. In this course, we
deal with exact values unless otherwise stated. For example, for sin 37◦ your calculator may show the value
0.60181502. But this is not the exact value, that is, sin 37◦ , 0.60181502. It is only an approximate value, i. e.,
sin 37◦ ≈ 0.60181502.
MATH1034 1st Semester Algebra Lecture Manual 2017 23

3.2.3 Revision: Trigonometric Periods


A function f is called periodic if there exist a positive number p such that x + p belongs to the domain of f if and
only x belongs to the domain of f and such that f (x + p) = f (x) for all x in the domain of f . The smallest such
value p (if it exists) is called the period of f .

The functions sin, cos, sec, cosec have period 2π.


The functions tan and cot have period π.

Note. If f is periodic with period p and x belongs to the domain of f , then for any integer n, x + np belongs to the
domain of f and f (x + np) = f (x).
25π
Worked Example 3.2.1. Using periodicity, evaluate sin .
6
Solution. Since
π
Å ã
25π 1
= 4+ π = (2)(2π) + ,
6 6 6
the periodicity of sin with period 2π gives

25π π  π 1
sin = sin + (2)(2π) = sin = .
6 6 6 2

3.2.4 Revision: Some Important Trigonometric Identities


Theorem 3.1.
1. sin(−θ) = − sin θ
cos(−θ) = cos θ
1 1
2. sec θ = cosec θ =
cos θ sin θ
sin θ cos θ
tan θ = cot θ =
cos θ sin θ
3. sin(θ + 2nπ) = sin θ for any integer n
cos(θ + 2nπ) = cos θ for any integer n
π 
sin − θ = cos θ
2
π 
cos − θ = sin θ
2
4. sin2 θ + cos2 θ = 1
1 + tan2 θ = sec2 θ
1 + cot2 θ = cosec2 θ

5. sin(θ ± ϕ) = sin θ cos ϕ ± cos θ sin ϕ


cos(θ ± ϕ) = cos θ cos ϕ ∓ sin θ sin ϕ
tan θ ± tan ϕ
tan(θ ± ϕ) =
1 ∓ tan θ tan ϕ

6. sin(2θ) = 2 sin θ cos θ


cos(2θ) = cos2 θ − sin2 θ
24 1st Semester Algebra Lecture Manual 2017 MATH1034

1 − cos 2θ
7. sin2 θ =
2
1 + cos 2θ
cos2 θ =
2
8. Product-to-sum formulae
1
sin θ cos ϕ = (sin(θ + ϕ) + sin(θ − ϕ))
2
1
cos θ cos ϕ = (cos(θ + ϕ) + cos(θ − ϕ))
2
1
sin θ sin ϕ = (cos(θ − ϕ) − cos(θ + ϕ))
2
9. Sum-to-product formulae
θ+ϕ θ−ϕ
Å ã Å ã
sin θ + sin ϕ = 2 sin cos
2 2
θ+ϕ θ−ϕ
Å ã Å ã
sin θ − sin ϕ = 2 cos sin
2 2
θ+ϕ θ−ϕ
Å ã Å ã
cos θ + cos ϕ = 2 cos cos
2 2
θ+ϕ θ−ϕ
Å ã Å ã
cos θ − cos ϕ = −2 sin sin
2 2

For the proof note that 1–4 are easy consequences of the definitions and the periodicity of the trigonometric func-
tions (the first identity in 4 is the Pythagorean Theorem). The first identity in 5 (with +) is proved by considering
suitable triangles (see Huntley/Love, Section 6.3) for the special case that all of θ, ϕ and θ + ϕ are positive and
less or equal π/2, and the remaining parts of the identities five then follows with the aid of the identities 1–3. The
identities in 6 and 7 are special cases of 5. The proofs of the identities 7 and 8 are done in Worked Examples 3.2.2
and 3.2.3 as well as in the tutorials.

Worked Example 3.2.2. Prove that sin θ cos ϕ = 12 (sin(θ + ϕ) + sin(θ − ϕ)).
Solution. From part 5 we know

sin(θ + ϕ) = sin θ cos ϕ + cos θ sin ϕ,


sin(θ − ϕ) = sin θ cos ϕ − cos θ sin ϕ.

Adding these two equations gives

sin(θ + ϕ) + sin(θ − ϕ) = 2 sin θ cos ϕ.

Dividing both sides by 2 and interchanging sides completes the proof.

θ+ϕ θ−ϕ
Å ã Å ã
Worked Example 3.2.3. Prove that sin θ + sin ϕ = 2 sin cos .
2 2
Solution. Let
θ+ϕ θ−ϕ
α= , β= .
2 2
Then
θ = α + β, ϕ = α − β,
and by Example 3.2.2,

sin θ + sin ϕ = sin(α + β) + sin(α − β)


= 2 sin α cos β
θ+ϕ θ−ϕ
Å ã Å ã
= 2 sin cos .
2 2
MATH1034 1st Semester Algebra Lecture Manual 2017 25


Worked Example 3.2.4. Evaluate sin .
12
Solution. Observe that
7π π π
= + .
12 3 4
Therefore
7π π π
sin = sin +
12 3 4
π π π π
= sin cos + cos sin
Ç √3 å Å4 ã 3Å ã4Å ã
3 1 1 1
= √ + √
2 2 2 2

1+ 3
= √ .
2 2

Theorem 3.1, 7., allows to write even powers of sin and cos in terms of powers of lower order of cos with multiple
angles. This is very useful in evaluating integrals (which will be done in the Calculus module).

Worked Example 3.2.5. Express sin4 θ in terms of cosine functions, without even powers.
Solution.
2
sin4 θ = sin2 θ (write the power in terms of sin2 )
1 − cos(2θ) 2
Å ã
= (apply Theorem 3.1, 7.)
2
1
= 1 − 2 cos(2θ) + cos2 (2θ)

(repeat, where necessary)

1 + cos(4θ)
ã
1
= 1 − 2 cos(2θ) +
4 2
3 1 1
= − cos(2θ) + cos(4θ).
8 2 8
Worked Example 3.2.6. Prove that
1
(a) sin2 x cos2 x = (1 − cos 4x).
8
sin 3x − sin x
(b) = tan x.
cos 3x + cos x
Solution. (a)
Å ã2
1
(sin x cos x) =
2
sin(2x) (by Theorem 3.1, 6.)
2
1
= sin2 (2x)
4Å ã
1 1 − cos(4x)
= (by Theorem 3.1, 7.)
4 2
1
= (1 − cos(4x)).
8
(b)
3+1 3−1
 
sin 3x − sin x 2 cos 2 x sin 2 x
= (by Theorem 3.1, 9.)
cos 3x + cos x 2 cos 3+1
2 x cos
3−1
2 x
sin x
=
cos x
= tan x.
26 1st Semester Algebra Lecture Manual 2017 MATH1034

Tutorial 3.2.1. 1. Prove the identities in Theorem 3.1, 8. and 9.


2. Determine the quadrant in which θ lies if
(a) sec θ > 0 and cot θ < 0,
(b) cosec θ < 0 and tan θ > 0.
1
3. Prove the identity cos3 θ sin θ = sin(4θ) + 14 sin(2θ).
8
θ
4. If r = tan , prove that
2
2r 2r 1 − r2
(a) tan θ = , (b) sin θ = , (c) cos θ = .
2
1π− r  1+ r 
2 1 + r2
π
5. Use sin − θ = cos θ and cos − θ = sin θ to prove that
2 2
(a) sin(θ + π) = − sin θ, (b) cos(θ + π) = − cos θ, (c) sin(θ + 2π) = sin θ, (d) cos(θ + 2π) = cos θ.
6. Calculate the following values:
Å ã

Å
11π
ã π Å
35π
ã
(a) sin , (b) cos , (c) tan , (d) sec .
12 12 12 12

3.3 Inverse Trigonometric Functions


Recall that a function has an inverse if and only if it is one-to-one on its domain. But the trigonometric functions
are not one-to-one (indeed, periodic functions are never one-to-one). However, for every of the six trigonometric
functions f we can find intervals I such that f restricted to I is one-to-one and such that the range of the restriction
of f to I equals the range of f . We will consider the inverses in this sense for the functions sin, cos, and tan.

3.3.1 The arcsine Function

Observe that range(sin) = [−1, 1], that sin is one-to-one on − π2 , π2 , and that the range of this restriction is [−1, 1]:
 

1 y = sin x

x
π π

2 2

−1

Here it is convenient to restrict the sine function to the interval − π2 , π2 since this interval is symmetric with respect
 

to 0 (and since the sine function is increasing there). We could also have taken the interval π2 , 3π
 
2 , for example.
 π π
 π π  − 2 , 2 has an inverse, called arcsine and denoted by arcsin. Hence arcsin has
The sine function on the interval
domain [−1, 1] and range − 2 , 2 . Since the graph of an inverse function is obtained from the original function
by reflection with respect to the line y = x, the graph of the arcsine function is easily obtained:
MATH1034 1st Semester Algebra Lecture Manual 2017 27

y
π
y = arcsin x
2

1 y = sin x

x
π −1 1 π

2 2

−1

π

2

Summing up, we have the following

Definition.

h π πi
For each x ∈ [−1, 1], y = arcsin x is the (unique) number y ∈ − , such that x = sin y.
2 2

Note. 1. An alternative notation for arcsin is sin−1 . However, we avoid this notation for the following two reasons:
The sine function is not invertible, only suitable restrictions are, and arcsin is the inverse of a particular restriction
of the sine function.
The notation may be somewhat ambiguous and may lead to confusion since we write sin2 x for (sin x)2 but sin−1 x
1
does not mean (sin x)−1 = .
sin x
2. Observe that sin(arcsin  x) = x for all x ∈ [−1, 1], i. e., for all x in the domain of arcsin, whereas arcsin(sin x) = x
if and only if x ∈ − π2 , π2 , i. e., if and only if x is in the range of arcsin.


3. It is useful to observe that arcsin is an odd function.

Worked Example 3.3.1. Find (if it exists):


Ç √ å
1 3
(a) arcsin (b) arcsin − (c) arcsin 2
2 2
Solution.
1 π
(a) arcsin =
2 6 å
√ Ç√ å
π
Ç
3 3
(b) arcsin − = − arcsin =−
2 2 3
(c) arcsin 2 is undefined since 2 is not in the domain of arcsin.
28 1st Semester Algebra Lecture Manual 2017 MATH1034

Worked Example 3.3.2.

  π  1 π
(a) arcsin sin = arcsin = (same angle)
6 2 6
1 π
Å Å ãã

(b) arcsin sin = arcsin = (not same angle)
6 2 6
  π  Å
1
ã
π
(c) arcsin sin − = arcsin − =− (same angle)
6 2 6

Note. Using the periodicity of sin and sin(±π− x) = sin x, sin x can easily be converted to sin x0 with x0 ∈ − π2 , π2 .
 

Worked Example 3.3.3. Find (if it exists):


  π  Å Å ãã

Å Å
37π
ãã
(a) arcsin sin − (b) arcsin sin (c) arcsin sin
9 8 7
Solution.
π h π πi   π  π
(a) Since − ∈ − , , arcsin sin − =− .
9 2 2 9 9
7π h π π i
(b) Observe that < − , , but that
8 2 2
7π π h π π i
π− = ∈ − ,
8 8 2 2
so that Å ã π

sin = sin
8 8
and Å Å ãã   π  π

arcsin sin = arcsin sin = .
8 8 8
37π
(c) Observe that < [−π, π], but that
7
37π 5π
− 6π = − ∈ [−π, π].
7 7
But
5π h π π i
− < − , .
7 2 2

Since − < 0 consider
7
2π h π π i
Å ã

−π − − =− ∈ − , .
7 7 2 2
Then Å ã Å ã Å ã
37π 5π 2π
sin = sin − = sin −
7 7 7
and Å Å ãã Å Å ãã
37π 2π 2π
arcsin sin = arcsin sin − =− .
7 7 7
Worked Example 3.3.4. Find (if it exists):
(a) sin(arcsin(0.8)) (b) sin(arcsin(−0.4)) (c) sin(arcsin(1.7))
Solution.
(a) sin(arcsin(0.8)) = 0.8
(b) sin(arcsin(−0.4)) = −0.4
(c) sin(arcsin(1.7)) does not exist since 1.7 does not belong to the domain of arcsin.
MATH1034 1st Semester Algebra Lecture Manual 2017 29

Tutorial 3.3.1. 1. Huntley/Love Exercises 5.6A pp. 584–585 (earlier edition pp. 336–337): 3, 6, 15, 20, 47.
Å ã
3 5
2. Evaluate cos arcsin + arcsin .
13 7
3. Show that arcsin(sin x) is periodic with period 2π and hence sketch this function.

3.3.2 The arccosine Function


Observe that range(cos) = [−1, 1], that cos is one-to-one on [0, π], and that the range of this restriction is [−1, 1]:

x
π π
2

−1 y = cos x

Note that the cosine function is decreasing on [0, π]. We could also have taken the interval [−π, 0], for example,
on which the cosine function is increasing.

Definition.

For each x ∈ [−1, 1], y = arccos x is the (unique) number y ∈ [0, π] such that x = cos y.

Note. 1. An alternative notation for arccos is cos−1 . The same remark as for arcsin also applies here.
2. Observe that cos(arccos x) = x for all x ∈ [−1, 1], i. e., for all x in the domain of arccos, whereas arccos(cos x) =
x if and only if x ∈ [0, π], i. e., if and only if x is in the range of arccos.

Worked Example 3.3.5. Find (if it exists):


Å ã
1 1
(a) arccos (b) arccos − √ (c) arccos(−3)
2 2
Solution.
1 π
(a) arccos =
2 3 ã
π 3π
Å Å ã
1 1
(b) arccos − √ = π − arccos √ =π− =
2 2 4 4
(c) arccos(−3) is undefined since −3 is not in the domain of arccos.

Worked Example 3.3.6.


Å Å ãã
5π 5π 5π
(a) arccos cos = (∵ ∈ [0, π])
6 6 6
π    π    π  π
Å Å ãã
23π  
(b) arccos cos = arccos cos 4π − = arccos cos − = arccos cos =
6 6 6 6 6

Note. Using the periodicity of cos and cos(−x) = cos x, cos x can easily be converted to cos x0 with x0 ∈ [0, π].
30 1st Semester Algebra Lecture Manual 2017 MATH1034

Worked Example 3.3.7. Verify that arccos(−x) = π − arccos x for all x ∈ [−1, 1].
Solution.
Let y = arccos x. Then y belongs to the range of arccos, i. e., y ∈ [0, π], and also π − y ∈ [0, π], and

cos(π − y) = − cos y = − cos(arccos x) = −x.

Since π − y ∈ [0, π], it follows that


π − y = arccos(−x),

or
arccos(−x) = π − arccos x.

Tutorial 3.3.2. 1. Huntley/Love Exercises 5.6A pp. 584–585 (earlier edition pp. 336–337): 4, 7, 16, 19, 48.
Å Å ã ã
3 5
2. Evaluate tan arcsin − − arccos .
5 13
3. Show that arccos(cos x) is periodic with period 2π and hence sketch this function.

3.3.3 The arctangent Function


Observe that range(tan) = R, that tan is one-to-one on − π2 , π2 , and that the range of this restriction is R:


y
y = tan x

x
π π

2 −1 2

Here it is convenient to restrict the tangent function to the interval − π2 , π2 since this interval is symmetric with

 π
respect to 0 (and since the tangent function is increasing there). We could also have taken the interval 3π 2 , 2 , for
example.

Definition.

 π π
For each x ∈ R, y = arctan x is the (unique) number y ∈ − , such that x = tan y.
2 2
MATH1034 1st Semester Algebra Lecture Manual 2017 31

The graph of arctan is given below.

y
π
2 y = arctan x
1

x
−1 1

−1
π

2

Note. 1. An alternative notation for arctan is tan−1 . The same remarks as for arcsin apply.
 x) = x for all x ∈ R, i. e., for all x in the domain of arctan, whereas arctan(tan x) = x if
2. Observe that tan(arctan
and only if x ∈ − π2 , π2 , i. e., if and only if x is in the range of arctan.
3. It is useful to observe that arctan is an odd function.

Worked Example 3.3.8. Find (if it exists):



(a) arctan 3 (b) arctan(−1)
Solution.
√ π π
(a) arctan 3= (b) arctan(−1) = − arctan 1 = −
3 4

Note. Using the periodicity of tan, tan x can easily be converted to tan x0 with x0 ∈ − π2 , π2 .


Worked Example 3.3.9. Find (if it exists):


  π  Å Å
13π
ãã
(a) arctan tan − (b) arctan tan
7 7
Solution.
π  π π   π  π
(a) Since − ∈ − , , arctan tan − =− .
7 2 2 7 7
13π  π π 
(b) Observe that < − , , but that
7 2 2
13π π  π π
− 2π = − ∈ − ,
7 7 2 2
so that Å ã  π
13π
tan = tan −
7 7
and   π  π
Å Å ãã
13π
arctan tan = arctan tan − =− .
7 7 7
Worked Example 3.3.10. Find tan(arctan 50)
Solution. tan(arctan 50) = 50

Note. For all x ∈ [−1, 1], arcsin x ∈ − π2 , π2 , and therefore


 

cos(arcsin x) ≥ 0,

and arccos x ∈ [0, π] leads to


sin(arccos x) ≥ 0.
32 1st Semester Algebra Lecture Manual 2017 MATH1034

Worked Example 3.3.11. Find


Å ã Å Å ãã Å ã
2 1 1 2
(a) cos arcsin (b) tan arccos − (c) cos arcsin − arccos
5 13 3 5
Å ã Å ã Å ã2 √
2 2 2 21
Solution. (a) cos arcsin = 1 − sin2 arcsin = 1− = .
5 5 5 5
Å Å
1
ãã √
2
1 − cos arccos − 168
13 √
Å Å ãã
1 13
(b) tan arccos − = = = − 168.
−1
Å Å ãã
13 1
cos arccos −
13 13
(c)
Å ã Å ã Å ã Å ã Å ã
1 2 1 2 1 2
cos arcsin − arccos = cos arcsin cos arccos + sin arcsin sin arccos
3 5 3 5 3 5
Å ãÅ ã Å ã Å ã
1 2 1 2
= 1 − sin2 arcsin + 1 − cos2 arccos
3 5 3 5
√ Å ã Å ã √
2 2 2 1 21
= +
3 5 3 5
√ √
4 2 + 21
= .
15
Tutorial 3.3.3. 1. Huntley/Love Exercises 5.6A pp. 584–585 (earlier edition pp. 336–337): 9, 10, 29, 31, 54.
2. Find x such that arctan 2 = arctan 4 − arctan x.
3. Show that arctan(tan x) is periodic with period π and hence sketch this function.

3.4 Trigonometric Equations


3.4.1 General Solutions to Trigonometric Equations

3
Suppose we seek all angles θ for which sin θ = .
2
π
Clearly, θ = is one particular solution, which lies in the first quadrant.
3
π
And θ = π − is another solution. It lies in the second quadrant.
3
As we see from the sketch below for a circle with centre (0, 0) and radius 1,

θ θ
x
ψ ψ

the equation sin θ = a for 0 < a < 1 has two solutions, one in the first quadrant and one in the second quadrant.
MATH1034 1st Semester Algebra Lecture Manual 2017 33

The equation sin ψ = b for 0 < b < 1 has two solutions, one in the third quadrant and one in the fourth quadrant.
π
The equation sin θ = 0 has two distinct solution θ = 0 and θ = π, whereas sin θ = ±1 has the solution ± .
2
More precisely, if x ∈ [−1, 1], then sin θ = x has exactly one solution in − π2 , π2 , and this solution is θ = arcsin x.
 

Similarly, there is exactly one solution in π2 , 3π


2 , and this solution is θ = π − arcsin x. Due to the periodicity of
 

sin, we therefore have:

If x ∈ [−1, 1], then sin θ = x if and only if there is k ∈ Z such that θ = arcsin x + 2kπ or θ = π − arcsin x + 2kπ.
If |x| > 1, then sin θ = x has no solution.

Similarly, if x ∈ [−1, 1], then cos θ = x has exactly one solution in [0, π], and this solution is θ = arccos x. Due
to cos(−x) = cos x, cos θ = x has exactly one solution in [−π, 0], and this solution is θ = − arccos x. Due to the
periodicity of sin, we therefore have:

If x ∈ [−1, 1], then cos θ = x if and only if there is k ∈ Z such that θ = arccos x + 2kπ or θ = − arccos x + 2kπ.
If |x| > 1, then cos θ = x has no solution.

Finally, for each x ∈ R, tan θ = x has exactly one solution in − π2 , π2 , and this solution is θ = arctan x. Due to the
 

periodicity of tan, we therefore have:

If x ∈ R, then tan θ = x if and only if there is k ∈ Z such that θ = arctan x + kπ.

Worked Example 3.4.1. Find the general solution to tan 5θ = −1


π
Solution. Since arctan(−1) = − arctan 1 = − ,
4
tan 5θ = −1 ⇔ 5θ = arctan(−1) + kπ for some k ∈ Z
π
⇔ 5θ = − + kπ for some k ∈ Z
4
π π
⇔ θ = − + k for some k ∈ Z.
20 5
Hence the general solution to tan 5θ = −1 is the set
n π π o
− +k k ∈Z .
20 5
Worked Example 3.4.2. Solve 2 cos2 θ − sin θ = 1.
Solution. First write this as an equation in sin θ:

2 cos2 θ − sin θ = 1 ⇔ 2 − 2 sin2 θ − sin θ = 1.

Putting x = sin θ gives the quadratic equation

2x2 + x − 1 = 0,

which has the solutions p


−1 ± (1)2 − 4(2)(−1) −1 ± 3
x= = ,
2(2) 4
1 π 1 π
and so sin θ = −1 or sin θ = . Since arcsin(−1) = − and arcsin = , the general solution is therefore
2 2 2 6
n π o nπ o ß 5π ™
− + 2kπ k ∈ Z ∪ + 2kπ k ∈ Z ∪ + 2kπ k ∈ Z .
2 6 6
34 1st Semester Algebra Lecture Manual 2017 MATH1034

To solve equations, it is often advantageous to write a sum as a product of simpler factors. Here the sum-to-product
formulae are useful.

Worked Example 3.4.3. Solve sin θ − cos 2θ = sin 3θ.


Solution.
3θ + θ 3θ − θ
Å ã Å ã
sin θ − cos 2θ = sin 3θ ⇔ − cos 2θ = sin 3θ − sin θ = 2 cos sin = 2 cos 2θ sin θ
2 2
⇔ −2 cos 2θ(1 + 2 sin θ) = 0
⇔ cos 2θ = 0 or (1 + 2 sin θ) = 0
π π 5π
⇔ 2θ = + kπ or θ = − + 2kπ or θ = − + 2kπ for some k ∈ Z.
2 6 6
Hence sin θ − cos 2θ = sin 3θ if and only if

π π π 5π
θ= + k or θ = − + 2kπ or θ = − + 2kπ for some k ∈ Z.
4 2 6 6
Worked Example 3.4.4. Solve sin 3θ = cos 5θ.
Solution.
π 
sin 3θ = cos 5θ ⇔ sin 3θ − sin − 5θ = 0
2
Ñ π é Ñ π é
3θ + − 5θ 3θ − − 5θ
⇔ 2 cos 2 sin 2 =0
2 2
π   π 
⇔ 2 cos − θ sin − + 4θ = 0
π4  4 
π 
⇔ cos − θ = 0 or sin − + 4θ = 0
4 4
π π π
⇔ − θ = + kπ or − + 4θ = kπ for some k ∈ Z
4 2 4
π π π
⇔ θ = − + kπ or θ = + k for some k ∈ Z.
4 16 4
Tutorial 3.4.1. 1. Solve the following trigonometric equations:
(a) 2 sec2 x − tan x = 3,
(b) sin x + sin 3x = sin x,
(c) cos 4x + cos 6x + 2 cos x = 0,
(d) cos x − 2 sin x cos x = cos 5x,
(e) 2 cos2 x = 3 cos 2x.

3.5 Polar Co-ordinates


3.5.1 Cartesian Co-ordinates and Polar Co-ordinates
We know that we can plot a point P(x, y) in the Cartesian plane by moving from the origin x units along the x-axis
and y units along the y axis. We call (x, y) the Cartesian co-ordinates of the point P.
There is another way of reaching the same point from the origin.
Starting at the origin and facing the positive x-axis we turn through an angle θ, anticlockwise, until we face the
point (x, y). We then cover the distance r until we reach the point (x, y). We call (r, θ) the polar co-ordinates of
the point (x, y).
MATH1034 1st Semester Algebra Lecture Manual 2017 35

y y
P(x, y) P(r, θ)

x θ x

Therefore we have
Definition (Polar co-ordinates).
The polar co-ordinates (r, θ) of a point P in the plane are defined as follows:

r is the (non-negative) distance of P from the origin;


θ is the (signed) angle between the positive x-axis and the the line from the origin to the point P.

Note. The polar coordinates of the point (x, y) are not unique since (r, θ) and (r, θ + 2nπ), n ∈ Z, represent the same
point.

Worked Example 3.5.1. (a) Use a sketch to find the polar co-ordinates of the point P with Cartesian co-ordinates
(x, y) = (0, 4).
(b) Use a sketch to find the Cartesian co-ordinates of the point Q with polar co-ordinates (r, θ) = (3, π).
π
Solution. (a) Clearly, θ = in this case, and r = y = 4.
2
(b) Clearly, Q lies on the negative real axis, an therefore (x, y) = (−3, 0).

Tutorial 3.5.1. 1. Plot the following points in the Cartesian plane:


Å ã Å ã
3π 7π
(a) (r, θ) = 4, , (b) (r, θ) = 9, ,
4 6
Å ã Å ã
5π 25π
(c) (r, θ) = 6, , (d) (r, θ) = 5, .
3 12

3.5.2 Converting Polar co-ordinates to Cartesian Co-ordinates and vice versa


In simple cases as in Example 3.5.1 it is easy to “see” the conversion between Cartesian and polar co-ordinates.
For general points P, the sketch

y
P(x, y)
y
r

θ x
x

and the definition of sin and cos show

x = r cos θ, y = r sin θ.
36 1st Semester Algebra Lecture Manual 2017 MATH1034

These formulae give x and y if r and θ are known.

To find r and θ if x and y are known, observe that

y
r2 = x2 + y2 (Pythagoras), tan θ = if x , 0.
x

y
Note. Since r ≥ 0, r is uniquely determined by r2 = x2 + y2 . However, tan θ = gives two values of θ in different
x
quadrants (if x , 0 and y , 0), and you have to either draw a sketch or look at the sign of x or y to find the correct
y y
quadrant. If x , 0, θ would be arctan or arctan + π.
x x

Worked Example 3.5.2. Find the polar coordinates of the point (x, y) = (−3, 3).
Solution.

r2 = x2 + y2 = 9 + 3 = 12 ⇒ r = 2 3.

y π
Å ã
3 1 1
tan = = − √ and arctan − √ =− .
x −3 3 3 6
π √
The angle − lies in the fourth quadrant, but (x, y) = (−3, 3) lies in the second quadrant. Hence we have to add
6
(or subtract) π to get the correct angle (observe that the angle is only unique up to adding integer multiples of 2π).
π 5π √ √ 5π
Å ã
Therefore θ = π − = . The polar coordinates of the point (x, y) = (−3, 3) are therefore (r, θ) = 2 3, .
6 6 6
Å ã

Worked Example 3.5.3. Find the Cartesian coordinates of the point with polar coordinates (r, θ) = 2, .
4
Solution.
5π √ 5π √
x = r cos θ = 2 cos =− 2 and y = r sin θ = 2 sin = − 2.
4 4
Tutorial 3.5.2. 1. Give polar co-ordinates for the following points in Cartesian co-ordinates:
√ √
(a) ( 3, −1), (b) (−2, 2), (c) (3, 3), (d) (0, 5).

3.5.3 Polar Equations


As we can convert Cartesian co-ordinates into polar co-ordinates and vice versa, we can also convert equations
in Cartesion coordinates into equations in polar co-ordinates and vice versa. Note that the polar coordinates (r, θ)
have the restriction r ≥ 0.

Worked Example 3.5.4. Convert the polar equation r = 2 to a Cartesian equation.


Solution.
r = 2 describes all points in the Cartesian plane which have a distance of 2 from the origin, i. e., a circle with
centre (0, 0) and radius 2. Indeed, since r = 2 if and only if r2 = 4 (note that r ≥ 0), the equation r = 2 in polar
co-ordinates is equivalent to the equation x2 + y2 = 4 in Cartesian co-ordinates.

Worked Example 3.5.5. Write x = 2 in polar form.


Solution.
x = 2 ⇔ r cos θ = 2.

Note. Some equations are easier in polar co-ordinates (like circles with centre at the origin), others easier in
Cartesian co-ordinates (like lines).
MATH1034 1st Semester Algebra Lecture Manual 2017 37

Worked Example 3.5.6. Convert the following equations into their Cartesian form:
(a) r = 2 sin θ, (b) r = 1 + cos θ.
Solution. (a)
r = 2 sin θ ⇒ r2 = 2r sin θ (multiplied by r)
⇔ x + y = 2y
2 2

⇔ x2 + (y − 1)2 = 1.
In the first step we have multiplied by r for easier conversion to Cartesian coordinates. This “adds” the points
with r = 0 as points satisfying the equation. But r = 0 ist just the origin, which satisfies the equation anyway (put
θ = 0).
(b) With a similar reasoning as in (a)
r = 1 + cos θ ⇔ r2 = r + r cos θ (multiplied by r)
p
⇔ x2 + y2 = x2 + y2 + x.
Tutorial 3.5.3. 1. Give an equation in polar co-ordinates for each of the following objects in the Cartesian plane:
(a) the circle with centre (0, 0) and radius 3;
(b) the circle with centre (1, 0) and radius 2;

(c) the circle with centre (a, b) and radius a2 + b2 ;
(d) the ray x = 0, y ≥ 0;
(e) the ray y = 0, x ≤ 0;
(f) the line x = 4;
(g) the line x + y = 1;
(h) the ray x = y, x > 0.
2. Transform the following equations from Cartesian to polar form, or vice versa.
(a) x2 − y2 = 1, (b) x2 + y2 = 2x,
(c) r = 3 cosec θ, (d) r = cos 2θ.

3.5.4 Polar Graphs


A polar graph is the set of all points in polar co-ordinates (r, θ) which satisfy an equation in terms of polar co-
ordinate variables.
Note. We have r ≥ 0 for polar co-ordinates (r, θ). So we will reject any values of θ that yield negative r.

Worked Example 3.5.7. Sketch the curve with polar equation r = 3.


Solution. The curve is a circle with centre (0, 0) and radius 3:

y
3

r=3

x
3
38 1st Semester Algebra Lecture Manual 2017 MATH1034

π
Worked Example 3.5.8. Sketch the curve with polar equation θ = .
4
Solution. The curve is a ray:

π
θ=
4
π
4 x

Worked Example 3.5.9. Sketch all points satisfying the polar inequalities 1 < r ≤ 3.
Solution. The set is an annulus:

x
1 3

π π
Worked Example 3.5.10. Sketch all points satisfying the polar inequalities <θ≤ .
3 2
Solution. The set is an unbounded sector:

Worked Example 3.5.11. Sketch the curve given in polar co-ordinates by r = θ.


Solution.

• Make a table of values (remember that r ≥ 0, so only θ ≥ 0 is allowed):


π π π 3π
θ 0 π 2π
4 3 2 2
r 0 0.79 1.05 1.57 3.14 4.71 6.28

• When θ increases, the curve turns anticlockwise and moves away from the origin. Hence we obtain a spiral.
• Plot some points and join them.
MATH1034 1st Semester Algebra Lecture Manual 2017 39

1 r=θ

x
−2 2 4 6
−1

−2

−3

−4

This curves is called Archimedes’ spiral.

Worked Example 3.5.12. Sketch the curve given in polar co-ordinates by r = −θ.

Solution. We again have a spiral, but here θ < 0. When θ decreases, the curve turns clockwise and moves away
from the origin. Hence

1 r=θ

x
−2 2 4 6
−1

Sketching polar graphs is greatly facilitated if one knows something about their symmetries. Hence, before we
start plotting graphs, we shall investigate the three different types of symmetries.
40 1st Semester Algebra Lecture Manual 2017 MATH1034

i) Symmetry about the x-axis

Take two points P and Q which are symmetrical about the x-axis. Then they have polar co-ordinate representations
P(r, θ) and Q(r, −θ):

P(r, θ)

θ
x
−θ

P(r, −θ)

That is, the points (r, θ) and (r, −θ) in polar co-ordinates are symmetrical about the x-axis.
Hence, a graph has symmetry about the x-axis if and only if for any (r, θ) satisfying the equation in polar co-
ordinates, also the point (r, −θ) satisfies the equation.
Thus we have this simple test: If, when replacing θ by −θ, the equation of the graph remains the same, then the
graph has symmetry about the x-axis.
For example, r = cos θ is symmetrical about the x-axis since cos(−θ) = cos θ.

ii) Symmetry about the y-axis

Take two points P and Q which are symmetrical about the y-axis. Then they have polar co-ordinate representations
P(r, θ) and Q(r, π − θ):

P(r, π − θ) P(r, θ)

π−θ

θ θ
x

That is, the points (r, θ) and (r, π − θ) in polar co-ordinates are symmetrical about the x-axis.
Hence, a graph has symmetry about the y-axis if and only if for any (r, θ) satisfying the equation in polar co-
ordinates, also the point (r, π − θ) satisfies the equation.
Thus we have this simple test: If, when replacing θ by π − θ, the equation of the graph remains the same, then the
graph has symmetry about the y-axis.
For example, r = sin 3θ is symmetrical about the y-axis since

sin(3(π − θ)) = sin(3π − 3θ) = sin(π − 3θ) = sin(3θ)

for all θ.
MATH1034 1st Semester Algebra Lecture Manual 2017 41

iii) Symmetry about the origin

Take two points P and Q which are symmetrical about the origin. Then they have polar co-ordinate representations
P(r, θ) and Q(r, π + θ):

y
P(r, θ)

π+θ
θ
x
θ

P(r, π + θ)

That is, the points (r, θ) and (r, π + θ) in polar co-ordinates are symmetrical about the x-axis.

Hence, a graph has symmetry about the origin if and only if for any (r, θ) satisfying the equation in polar co-
ordinates, also the point (r, π + θ) satisfies the equation.

Thus we have this simple test: If, when replacing θ by π + θ, the equation of the graph remains the same, then the
graph has symmetry about the origin.
For example, r = cos 2θ is symmetrical about the origin since

cos(2(π + θ)) = cos(2π + 2θ) = cos 2θ

for all θ.

We are now ready to plot polar graphs which have sin and cos terms.

Worked Example 3.5.13. Sketch the polar graph r = 2 cos(3θ).

Solution.

• Examine the polar equation for symmetry: 2 cos(3(−θ)) = 2 cos(−3θ) = 2 cos 3θ.

∴ The graph is symmetrical about the x-axis.

∴ We only have to plot points in the first and second quadrant.

• Examine the polar equation for periodicity:

2 cos(3(2π + θ)) = 2 cos(6π + 3θ) = 2 cos 3θ.

∴ We only have to consider θ ∈ [0, 2π].

∴ Taking the above symmetry into account, we only have to consider θ ∈ [0, π].

• Find where r ≥ 0:

For this it is convenient to plot the graph r = 2 cos 3θ in the θ-r plane:
42 1st Semester Algebra Lecture Manual 2017 MATH1034

r
r = 2 cos 3θ
2

θ
π π π 2π 5π π
6 3 2 3 6
−1

−2

h π i ï π 5π ò
We deduce for θ ∈ [0, π] that r ≥ 0 when θ ∈ 0, ∪ , .
6 2 6
• Set up a table of values, for θ in the above set, or read them off from the above graph:
π π π 7π 2π 3π 5π
θ 0
12 6 2 12 3 4 6
r = 2 cos 3θ 2 1.41 0 0 1.41 2 1.41 0

We can now sketch the graph of r = 2 cos(3θ).

1.5

1 r = 2 cos 3θ

0.5

x
−1 −0.5 0.5 1 1.5 2
−0.5

−1

−1.5

The name for this type of a polar graph is a rose with 3 petals.

Worked Example 3.5.14. Sketch the polar graph r = 1 + sin θ.


Solution. Since
1 + sin(π − θ) = 1 + sin θ,
the hgraph isi symmetrical about the y-axis. Also, the function is 2π peridic. Hence it is sufficient to consider
π π π π
θ ∈ − , . At θ = − , r = 0, if θ increases to 0, r increases to 1, and if θ increases further to , then r increases
2 2 2 2
to 2. This gives the following shape:
MATH1034 1st Semester Algebra Lecture Manual 2017 43

1.5 r = 1 + sin θ

0.5

x
−1.5 −1 −0.5 0.5 1 1.5
−0.5

This type of graph is called a cardioid (it has the shape of a heart).

3.5.5 Questions for you to investigate:


i) How many petals does the graph of r = 5 cos 4θ have? Sketch the graph.
ii) What would the graph of r = b cos nθ look like?
a) What would happen if you allowed r to be negative?
b) Would it make a difference if n was even or odd?

3.5.6 Summary
(a) Polar curves of the form r = c, c > 0, give a circle with centre at the origin, radius c. See Example 3.5.7.
(b) Curves of the form θ = k give a ray starting at the origin. See Example 3.5.8.
(c) Polar curves of the form r = kθ give a spiral.
To plot curves of this form
• find r for easy values of θ (use a table).
• consider whether r increases or decreases as θ increases (this helps you establish a pattern when you plot
points).
• plot (r, θ) until you see a pattern emerging.
See Examples 3.5.11, 3.5.12.
(d) Polar curves of the form r = a + b sin kθ or r = a + b sin kθ give a rose, circle or cardioid.
To plot curves of this form
• examine the polar equation for symmetry, if any.
• find where r ≥ 0 (This can be done by plotting the graph of r = a + b sin kθ or r = a + b sin kθ considering
θ and r as Cartesian co-ordinates.
• find the values of θ at which r attains its maximum and its minimum (you can use the Cartesian graph
for this).
• you may set up a table of values using the above information to guide you in your choice of θ.
See Examples 3.5.13, 3.5.14.
44 1st Semester Algebra Lecture Manual 2017 MATH1034

Tutorial 3.5.4. 1. Sketch the regions in the x-y plane given in polar co-ordinates by
√ 4π π 2π
(a) r = 5, (b) θ = , (c) 3 ≤ r ≤ 4, (d) ≤ θ ≤ .
3 6 3
2. Sketch in the x-y plane the following curves given in polar co-ordinates:
π π
(a) r = cos θ, − ≤ θ ≤ , (b) r = 3 sin θ, (c) r = 1 + cos θ
2 2
(d) r = sin 3θ, (e) tan θ = −1, (f) rθ = π, θ > 0,
|θ|
(g) r = cos2 θ, (h) r = 1 + , −π ≤ θ ≤ π.
π

3.6 Expressions of the Form a cos x + b sin x


In this section we will show how to convert an expression of the form a cos x + b sin x into an expression of the
form R cos(x − θ).
First notice that

R cos(x − θ) = R cos θ cos x + R sin θ sin x


= a cos x + b sin x

by setting a = R cos θ and b = R sin θ.


That is, we can regard (a, b) as the Cartesian co-ordinates of the point with polar co-ordinates (R, θ).
We can use this in reverse: Any expression of the form

a cos x + b sin x

can be written in the form


R cos(x − θ)
where (R, θ) are the polar co-ordinates of the point (a, b) in Euclidean co-ordinates:

a = R cos θ and b = R sin θ.

Now we may use the methods derived in Subsection 3.5.2 to find R and θ from a and b.

Worked Example 3.6.1. Express cos x + 3 sin x in the form R cos(x − θ).
√ √
Hence solve the equation cos x + 3 sin x = 2.
Solution. Writing

cos x + 3 sin x = a cos x + b sin x

we have a = 1 and b = 3. Therefore


Ä √ ä2
R= a2 + b2 = 12 + 3 =2

and
b √
tan θ = = 3.
a
Since (a, b) lies in the first quadrant, we get
√ π
θ = arctan 3= .
3
Therefore √  π
cos x + 3 sin x = R cos(x − θ) = 2 cos x − ,
3
MATH1034 1st Semester Algebra Lecture Manual 2017 45

√ √
and cos x + 3 sin x = 2 can be written as

 π √  π 2
2 cos x − = 2 or cos x − = ,
3 3 2
which has the solutions √
π 2 π
x − = arccos + 2kπ = + 2kπ
3 2 4
and √
π 2 π
x − = − arccos + 2kπ = − + 2kπ
3 2 4
√ √
with k ∈ Z. Therefore, the the general solution of cos x + 3 sin x = 2 is the set of all x given by
7π π
x= + 2kπ or x = + 2kπ, k ∈ Z.
12 12
Tutorial 3.6.1. 1. Find the general solution of each of the following trigonometric equations:

3 √ 3 √
(a) − cos x + sin x = , (b) sin2 x − 3 sin x cos x = , (c) 2 cos x − sin x = 3.
2 2
2. Sketch the curves defined by

(a) y = cos x − sin x, (b) y = cos x + 3 sin x.
√  π
3. Find all values of x such that 12 cos 4x − 2 sin 4x = 2 sin 8x + .
3
Chapter 4

Mathematical Induction

LEARNING OUTCOMES:
On completion of this chapter you should (tick the checkbox when you have mastered the skill)
 1. understand the principle of mathematical induction,
 2. be able prove statements by mathematical induction,
 3. know the summation formulae for arithmetic and geometric progressions,
 4. know how to solve the examinable tutorial problems and the worked out problems from these notes on
this chapter.

4.1 Introduction
1
Suppose you are asked to prove that 12 + 22 + 32 + 42 + · · · + n2 = n(n + 1)(2n + 1) for any positive integer n.
6
You might start by verifying that the statement is true for different values of n.
When n = 1,
1
LHS: 12 = 1, RHS: (1)(1 + 1)(2 + 1) = 1
6
∴ statement is true for n = 1.
When n = 2,
1
LHS: 12 + 22 = 5, RHS: (2)(2 + 1)(4 + 1) = 5
6
∴ statement is true for n = 2.
When n = 3,
1
LHS: 12 + 22 + 32 = 14, RHS: (3)(3 + 1)(6 + 1) = 14
6
∴ statement is true for n = 3.
We could carry on testing this statement for many, many values of n, but you will never finish your task—there
will be (infinitely) more numbers for which you will need to test the statement.
Thus we ask if there is any way in which we can prove that this statement is true for all natural numbers.
Indeed, this statement can be proved by a method called mathematical induction. The oldest documented use
of this method can be found in works of ancient Greek mathematicians. However, a rigorous formulation of
the principle of mathematical induction was only established in the 19th century. The principle of mathematical
induction is equivalent to Peano’s axiom, see Subsection A.2.1.

46
MATH1034 1st Semester Algebra Lecture Manual 2017 47

4.2 The Principle of Mathematical Induction

Suppose we have a statement associated with each n ∈ N. If


(i) the statement is true when n = 1, and
(ii) whenever the statement is true for n = k, it is also true for n = k + 1,
then the statement is true for all n ∈ N.

Note. Since the principle of mathematical induction is equivalent to Peano’s axiom on the existence of the natural
numbers, it is also called the axiom of mathematical induction.
As with the existence of natural numbers, the principle of mathematical induction appears to be reasonable. In-
deed, we get the following:

• We estasblish the truth for n = 1 (the basis), see (i).

• if we have established the induction step, that is (ii), then

– truth for n = 2 follows since we have shown that the statement is true for n = 1,
– truth for n = 3 follows since we have shown that the statement is true for n = 2,
– truth for n = 4 follows since we have shown that the statement is true for n = 3,
– and so on.

So if (i) and (ii) hold, the statement must be true for all n ∈ N.

More formally and concisely, the principle of mathematical induction can be stated as follows:

For all n ∈ N let P(n) be a statement. If


(i) P(1) is true
and if
(ii) P(k) is true ⇒ P(k + 1) is true holds for all k ∈ N,
then P(n) is true for all n ∈ N.

Worked Example 4.2.1. Use mathematical induction to prove that 1 + 3 + 5 + · · · + (2n − 1) = n2 for all n ∈ N,
i. e., prove that the sum of the first n odd numbers is n2 .
Solution.
1
Worked Example 4.2.2. Use mathematical induction to prove that 12 + 22 + 32 + 42 · · · + n2 = n(n + 1)(2n + 1)
6
for all n ∈ N.
1
Solution. Let P(n) be the statement 12 + 22 + 32 + 42 · · · + n2 = n(n + 1)(2n + 1).
6

1
• P(1) is the statement 1 = (1)(1 + 1)(2(1) + 1), which is indeed true.
6
• We need to show that P(k + 1) is true whenever P(k) is true.

1
Assume P(k) is true, i. e., suppose that 12 + 22 + 32 + 42 · · · + k2 = k(k + 1)(2k + 1). (1)
6
48 1st Semester Algebra Lecture Manual 2017 MATH1034

Now P(k + 1) is the statement that


1
12 + 22 + 32 + 42 · · · + k2 + (k + 1)2 =
(k + 1)((k + 1) + 1)(2(k + 1) + 1). (2)
6
Making use of the induction hypothesis (1) we have

LHS of (2) = 12 + 22 + 32 + 42 · · · + k2 + (k + 1)2


1
= k(k + 1)(2k + 1) + (k + 1)2 (using (1))
6
1
= (k + 1)[k(2k + 1) + 6(k + 1)]
6
1
= (k + 1)(2k2 + k + 6k + 6)
6
1
= (k + 1)(2k2 + 7k + 6). (3)
6
On the other hand,
1
RHS of (2) = (k + 1)(k + 2)(2k + 3)
6
1
= (k + 1)(2k2 + 7k + 6). (4)
6
(3) and (4) now show that LHS of (2) = RHS of (2), i. e., P(k + 1) is true.
So both conditions of the principle of mathematical induction hold, i. e.,

• P(1) is true,

• P(k) is true ⇒ P(k + 1) is true.

Hence, by the principle of mathematical induction, P(n) is true for all n ∈ N.


Thus we have shown that 1 + 3 + 5 + · · · + (2n − 1) = n2 for all n ∈ N.

Worked Example 4.2.3. Prove by induction that 5n − 4n − 1 is divisible by 16.


Solution.
First let n = 1. Then 5n − 4n − 1 = 51 − 4(1) − 1 = 0 is divisible by 16. So the statement is true for n = 1.
Assume that the statement is true for k, i. e., suppose that 5k − 4k − 1 is divisible by 16. (1)
Then

5k+1 − 4(k + 1) − 1 = (5)5k − 4k − 5


= 5(5k − 4k − 1) + 5(4k + 1) − 4k − 5
= 5(5k − 4k − 1) + 20k + 5 − 4k − 5
= 5(5k − 4k − 1) + 16k.

Clearly, 16k is divisible by 16, and 5(5k − 4k − 1) is divisible by 16 by (1).


Hence, the statement is true for k + 1.
Thus we have shown that 5n − 4n − 1 is divisible by 16.

Worked Example 4.2.4. Prove by induction that 1 + 2n ≤ 3n for all n ∈ N.


Solution. Let P(n) be the statement 1 + 2n ≤ 3n for all n ∈ N.

• P(1) is the statement 3 ≤ 3, which is indeed true.

• We need to show that P(k + 1) is true whenever P(k) is true.


MATH1034 1st Semester Algebra Lecture Manual 2017 49

Assume P(k) is true, i. e., suppose that 1 + 2k ≤ 3k . (1)


Then

1 + 2(k + 1) = 1 + 2k + 2
≤ 3k + 2 (using (1))
≤ 3 + 2(3 )
k k
(since 3k ≥ 31 ≥ 1)
= (1 + 2)3k = 3k+1 ,

which shows that P(k + 1) is true.


Hence by the principle of mathematical induction, P(n) is true for all n ∈ N.
Thus we have shown that 1 + 2n ≤ 3n for all n ∈ N.

Note. Proof by induction need not start with n = 1. Some claims may only be true for larger values of n, say
n ≥ 4, not n ≥ 1. To deal with this situation, we can state the priciple of mathematical induction as follows:

Let m ∈ N and for all n ∈ N, n ≥ m, let P(n) be a statement. If


(i) P(m) is true
and if
(ii) P(k) is true ⇒ P(k + 1) is true holds for all k ∈ N, k ≥ m,
then P(n) is true for all n ∈ N, n ≥ m.

1 1 1 1 n−2
Worked Example 4.2.5. Prove by induction that + + +···+ = for all n ∈ N with
2·3 3·4 4·5 (n − 1)n 2n
n ≥ 3.
1 3−2
Solution. For n = 3, the statement is = , which is true.
2·3 2·3
1 1 1 1 k−2
Now let k ≥ 3 and assume the statement is true for k: + + + ··· + = . (1)
2·3 3·4 4·5 (k − 1)k 2k
Then
1 1 1 1 1 k−2 1
+ + + ··· + + = + (using (1))
2·3 3·4 4·5 (k − 1)k ((k + 1) − 1)(k + 1) 2k k(k + 1)
(k − 2)(k + 1) + 2
=
2k(k + 1)
(k2 − k − 2) + 2
=
2k(k + 1)
k(k − 1)
=
2k(k + 1)
k−1
=
2(k + 1)
(k + 1) − 2
= ,
2(k + 1)
which shows that the statement is true for k + 1.
1 1 1 1 n−2
Hence by the principle of mathematical induction, we have shown that + + +· · ·+ =
2·3 3·4 4·5 (n − 1)n 2n
for all n ∈ N with n ≥ 3.

We now use induction to prove the following summation formulas:


50 1st Semester Algebra Lecture Manual 2017 MATH1034

Theorem 4.1. 1. Let a, d ∈ R. Then we have the following summation formula for the arithmetic progression:
n
a + (a + d) + (a + 2d) + (a + 3d) + · · · + (a + (n − 1)d) = [2a + (n − 1)d] for all n ∈ N.
2
2. Let a, r ∈ R, r , 1. Then we have the following summation formula for the geometric progression:
a(1 − rn )
a + ar + ar2 + ar3 + · · · + arn−1 = for all n ∈ N.
1−r
1
Proof. 1. For n = 1, the statement is a = [2a], which is true.
2
For the induction step assume the statement is true for k ∈ N:
k
a + (a + d) + (a + 2d) + (a + 3d) + · · · + (a + (k − 1)d) = [2a + (k − 1)d]. (1)
2
We have to prove that the statement is true for k + 1:
k+1
a + (a + d) + (a + 2d) + (a + 3d) + · · · + (a + (k − 1)d) + (a + kd) = [2a + kd]. (2)
2
We calculate
LHS of (2) = a + (a + d) + (a + 2d) + (a + 3d) + · · · + (a + (k − 1)d) + (a + kd)
k
= [2a + (k − 1)d] + (a + kd) (using (1)
2
k
= (ka + a) + [(k − 1)d + 2d]
2
k
= (k + 1)a + (k + 1)d
ï 2 ò
k
= (k + 1) a + d
2
k+1
= [2a + kd]
2
= RHS of (2).

Hence the arithmetic progression formula is proved by the principle of mathematical induction.
1−r
2. For n = 1, the statement is a = a , which is true.
1−r
For the induction step assume the statement is true for k ∈ N:
a(1 − rk )
a + ar + ar2 + ar3 + · · · + ark−1 = . (3)
1−r
We have to prove that the statement is true for k + 1:
a(1 − rk+1 )
a + ar + ar2 + ar3 + · · · + ark−1 + ark = . (4)
1−r
We calculate
LHS of (4) = a + ar + ar2 + ar3 + · · · + ark−1 + ark
a(1 − rk )
= + ark (using (3))
1−r
a(1 − rk ) + ark (1 − r)
=
1−r
a − ark + ark − arrk
=
1−r
a − ark+1
=
1−r
a(1 − rk+1 )
=
1−r
= RHS of (4).
MATH1034 1st Semester Algebra Lecture Manual 2017 51

Hence the geometric progression formula is proved by the principle of mathematical induction.

Tutorial 4.2.1. 1. Use mathematical induction to prove the following statements:


a) 2 + 4 + 6 + 8 + · · · + 2n = n(n + 1) for all n ∈ N.
b) 5 + 7 + 9 + 11 + · · · + (2n + 1) = n2 + 2n − 3 for all n ∈ N.
c) n3 − n is divisible by 3 for all n ∈ N.
d) 2n > n2 if n ∈ N and n is large enough.
n2 (n + 1)2
e) 13 + 23 + 33 + 44 + · · · + n3 = for all n ∈ N.
4
f) 7n − 1 is divisible by 6 for all n ∈ N.
Chapter 5

Sigma Notation and Binomial Theorem

LEARNING OUTCOMES:
On completion of this chapter you should (tick the checkbox when you have mastered the skill)

 1. understand and know how to write sigma notation,


 2. be able to write a sum in sigma notation,
 3. know and understand telescoping series,
 4. know and understand the definition of factorials and binomial coefficients,
 5. know and understand Pascal’s triangle,
 6. know and understand the statement and the proof of the binomial theorem,
 7. be able to expand powers of sums using the binomial theorem,
 8. know how to solve the examinable tutorial problems and the worked out problems from these notes on
this chapter.

5.1 Sigma Notation


5.1.1 Introduction to Sigma Notation
Sigma notation gives us a concise and mathematically
P exact way of writing sums. It is called sigma notation
because we use the Greek capital letter sigma, , to denote ‘the sum of’.

n
X
The sum of n terms a1 , a2 , a3 , . . . , an is written as a j.
j=1
n
X
So a j means a1 + a2 + a3 + · · · + an .
j=1

a j is called the jth term of the sum.


The subscript j is called the index of summation and runs through the integers from 1 to n.

52
MATH1034 1st Semester Algebra Lecture Manual 2017 53

8
X
Worked Example 5.1.1. (a) j means 3 + 4 + 5 + 6 + 7 + 8.
j=3
6
X
(b) 2k = 21 + 22 + 23 + 24 + 25 + 26 .
k=1

Note. The index of summation is called a dummy variable since any symbol can be used for it. However, we will
usually use the letters i, j, k, or r.

Worked Example 5.1.2. The following sums are given in sigma notation. Write out these sums, term by term,
without using sigma notation. You do not need to evaluate the sums.
8
X 6
X 10
X n
X
(a) k3 , (b) (2i − 5), (c) 3, (d) 2 j.
k=3 i=1 k=1 j=1

8
X
Solution.(a) k3 = 33 + 43 + 53 + 63 + 73 + 83 ,
k=3
6
X
(b) (2i − 5) = (2 − 5) + (4 − 5) + (6 − 5) + (8 − 5) + (10 − 5) + (12 − 5),
i=1
10
X
(c) 3 = 3 + 3 + 3 + 3 + 3 + 3 + 3 + 3 + 3 + 3,
k=1
Xn
(d) 2 j = 2 + 4 + 6 + · · · + 2n.
j=1

Worked Example 5.1.3. Express the following sums in sigma notation.


(a) 3 + 6 + 9 + 12 + · · · + 30,
(b) −6 − 4 − 2 + 0 + 2 + 4 + · · · + 20,
(c) 2 + 4 + 6 + 8 + · · · + 2n,
2 3 4
(d) 1 + + + + . . . (n terms).
3 5 7
10
X
Solution. (a) 3 + 6 + 9 + 12 + · · · + 30 = 3 j,
j=1
10
X 14
X
(b) −6 − 4 − 2 + 0 + 2 + 4 + · · · + 20 = 2j or −6 − 4 − 2 + 0 + 2 + 4 + · · · + 20 = (2 j − 8),
j=−3 j=1
n
X
(c) 2 + 4 + 6 + 8 + · · · + 2n = 2k,
k=1
n
2 3 4 X j
(d) 1 + + + + ··· = (n terms).
3 5 7 j=1
2j − 1

In order to be able to prove results involving sums of variable length, we need the following formal definition of
sums involving sigma notation.
54 1st Semester Algebra Lecture Manual 2017 MATH1034

Definition. Let r ∈ N. Then we define


r
X
ai = ar ,
i=r

and inductively, for n ≥ r,


n+1
X n
X
ai = ai + an+1 .
i=r i=1

Theorem 5.1. Let m, n, r be integers with r ≤ n or r ≤ m < n, respectively, and let k be a constant. Then
n
X n
X n
X
1. (ai + bi ) = ai + bi ,
i=r i=r i=r

n
X n
X
2. kai = k ai ,
i=r i=r

m
X n
X n
X
3. ai + ai = ai ,
i=r i=m+1 i=r

n
X
4. 1 = n − r + 1,
i=r

n
X
5. k = (n − r + 1)k.
i=r

Proof. 1. We are using mathematical induction. For n = r the statement reads (ar + br ) = ar + br , which is
obviously true.
Now let k ∈ N, k ≥ r, such that the formula is true for k:
k
X k
X k
X
(ai + bi ) = ai + bi . (1)
i=r i=r i=r

Then
k+1
X k
X
(ai + bi ) = (ai + bi ) + (ak+1 + bk+1 ) (by definition of the sum)
i=r i=r
k
X k
X
= ai + bi + ak+1 + bk+1 (using (1))
i=r i=r
k
! k
!
X X
= ai + ai+1 + bi + bk+1 (associative law)
i=r i=r
k+1
X k+1
X
= ai + bi , (by definition of the sum)
i=r i=r

which is the formula for k + 1.


3. Here we use induction for n > m. The induction base is n = m + 1. We have
m
X m+1
X m
X m+1
X
ai + ar = ai + am+1 = ai ,
i=r i=m+1 i=r i=r

which proves the induction base.


MATH1034 1st Semester Algebra Lecture Manual 2017 55

For the induction step assume the statement is true for n = k for some k > m. Then
m k+1 m k
!
X X X X
ai + ai = ai + ai + ak+1 (by definition of the sum)
i=r i=m+1 i=r i=m+1
m k
!
X X
= ai + ai + ak+1 (associative law)
i=r i=m+1
k
X
= ai + ak+1 (by induction hypothesis)
i=r
k+1
X
= ai (by definition of the sum).
i=r

2, 4, 5: See Tutorials.

Tutorial 5.1.1. 1. Write the following in sigma notation:


(a) 4 + 9 + 14 + 19 + · · · + 74, (b) 33 + 34 + 35 + · · · + 32n ,
2 4 2n
(c) 2 − 3 + 4 − 5 + · · · − (2n + 1), (d) + + ··· + .
5 7 2n + 3
2. Prove parts 2, 4, 5 of Theorem 5.1.
3. Use mathematical induction to prove that
n
X 1 n
= .
j=1
j( j + 1) n + 1

5.1.2 Revision of Formulae for Some Simple Sums


Using sigma notation, the arithmetic and geometric series formulae in Theorem 4.1 can be written as

Corollary 5.2. For all a, d, r ∈ R, r , 1, and n ∈ N, we have the following summation formulae:
n−1
X n
1. Arithmetic series: (a + jd) = [2a + (n − 1)d],
j=0
2
n−1
X a(1 − rn )
2. Geometric series: ar j = .
j=0
1−r

Worked Example 5.1.4. Evaluate the following sums:


n n Å ã
X X 1
(a) (3r + 5), (b) 2 + r .
r

r=1 r=0
3
Solution. (a) This is an arithmetic series with n terms, a = 3 + 5 (note a is the first term of the series) and d = 3.
Hence
n
X n n
(3r + 5) = [2(8) + (n − 1)3] = [3n + 13].
r=1
2 2

1
(b) This is a sum of two geometric series with n + 1 terms, a = 1 and r = 2 and r = , respectively. Hence
3
Å ãn+1
1
n Å n n n+1 1−
3n+1 − 1
ã X
X 1 X 1 1 − 2 3
2r + r = 2r + = + = 2n+1
− 1 + .
3 3r 1−2 1 2 · 3n
r=0 r=0 r=0 1−
3
56 1st Semester Algebra Lecture Manual 2017 MATH1034

Next we will investigate how to find sums of the first n positive integer, sums of the squares and cubes of the first
n integers, and so one. That is, we will attempt to find
n
X
rm for integers m, n.
r=1

We will give formulas for m = 1, 2, 3. The case m = 1 is a special case of an arithmetic series, so we already know
the result.

We will, however, give a constructive procedure to find the sums, which allows, in principle, to find formulas for
all m.

m = 1: In this case, we write the series twice, once forward and once backward and add up both series:

1 + 2 + 3 + ... + n−2 + n−1 + n


n + n−1 + n−2 + ... + 3 + 2 + 1 +

n+1 + n+1 + n+1 + ... + n+1 + n+1 + n+1

This gives
n
X n
X
2 r= (n + 1) = n(n + 1) by Theorem 5.1, 5.
r=1 r=1

Hence
n
X n(n + 1)
r= . (1)
r=1
2

n
X
For m > 1, this simple approach does not work. One could add up the values of rm for some values until one
r=1
can see a pattern and thus conjecture a formula. This formula then has to be proved by mathematical induction.
n
X
However, there is a more systematic way. Suppose we already know summation formulae for rk with k < m.
r=1
Then we use the following “trick”. Start with

(n + 1)m+1 − 1,

and subtract and add all powers rm+1 in between. Thus we have

(n + 1)m+1 − 1 = (n + 1)m+1 − nm+1 + nm+1 − · · · − 4m+1 + 4m+1 − 3m+1 + 3m+1 − 2m+1 + 2m+1 − 1
= ((n + 1)m+1 − nm+1 ) + (nm+1 − (n − 1)m+1 ) + · · · + (4m+1 − 3m+1 ) + (3m+1 − 2m+1 ) + (2m+1 − 1)
X n
= ((r + 1)m+1 − rm+1 ),
r=1

which is called a telescoping series since it “folds up” into just two terms.
How does this general formula
n
X
(n + 1)m+1 − 1 = ((r + 1)m+1 − rm+1 ) (2)
r=1
n
X
enable us to find rm ?
r=1
Consider now m = 2. Then

(r + 1)3 − r3 = r3 + 3r2 + 3r + 1 − r3 = 3r2 + 3r + 1,


MATH1034 1st Semester Algebra Lecture Manual 2017 57

and (2) gives

n
X
(n + 1)3 − 1 = (3r2 + 3r + 1)
r=1
X n n
X n
X
=3 r2 + 3 r+ 1
r=1 r=1 r=1
n
X n(n + 1)
=3 r2 + (3) + n.
r=1
2

Solving for the unknown sum gives

n
3n(n + 1)
ï ò
X 1
r2 = (n + 1)3 − 1 − n −
r=1
3 2
1
= (n + 1) 2(n + 1)2 − 2 − 3n
 
6
1
= (n + 1) 2(n2 + 2n + 1) − 2 − 3n
 
6
1
= (n + 1) 2n2 + n
 
6
n
= (n + 1)(2n + 1).
6

These results, together with the case for m = 3, which is to be proved in the tutorials, are summarised in

Theorem 5.3. For all n ∈ N:


n
X n(n + 1)
1. r= ,
r=1
2
n
X n
2. r2 = (n + 1)(2n + 1),
r=1
6
n
X n2 (n + 1)2
3. r3 = .
r=1
4

n Å ã
X 1 1
Worked Example 5.1.5. Evaluate − .
r=2
r−1 r

Solution. This is a telescoping series:

n Å ã Å ã Å ã Å ã
X 1 1 1 1 1 1 1 1
− = 1− + − + ··· + − =1− .
r=2
r−1 r 2 2 3 n−1 n n

Alternatively, we can provide a more formal proof using sigma notation if we observe the following index shift
formula for all integers k ≤ n and m:

n
X n+m
X
ar = ar−m ,
r=k r=k+m
58 1st Semester Algebra Lecture Manual 2017 MATH1034

because both sums equal ak + ak+1 + · · · + an . Hence


n Å ã X n n
X 1 1 1 X 1
− = −
r=2
r − 1 r r=2
r − 1 r=2
r
n−1 n
X 1 X 1
= − (index shift)
r=1
r r=2
r
n−1 n
!
X 1 X 1 1
= +1− +
r=2
r r=2
r n
1
=1− .
n
19
X
Worked Example 5.1.6. Evaluate r(r + 2).
r=5

Solution.
19
X 19
X
r(r + 2) = (r2 + 2r)
r=5 r=5
19
X 4
X
= (r + 2r) −
2
(r2 + 2r)
r=1 r=1
19
X 19
X 4
X 4
X
= r2 + 2 r− r2 − 2 r
r=1 r=1 r=1 r=1
19 4
= (20)39 + 19(20) − (5)(9) − 4(5)
6 6
= 190(13 + 2) − 30 − 20
= 2800.

Tutorial 5.1.2. 1. Prove, without the use of mathematical induction, that


n
X n2 (n + 1)2
r3 = .
r=1
4

2. Evaluate the following sums


n Å ã n Å ã n
X 1 1 X 1 2 1 X
(a) − , (b) − + , (c) (−1)r .
r=6
2r + 1 2r − 1 r=1
r r+1 r+2 r=1
3. Finds what should replace the question marks in the following equations:
n+3 ? 2n ? ?
X 1 X 1 X 2 X1 X 1
(a) = (b) = + .
2r − 4 2r r r r=? 1
r=5 r=? r=4 r=? r+
2
4. Evaluate the following sums:
(a) 10 + 17 + 26 + 37 + 50 + . . . (n terms),
(b) 4 · 3 + 9 · 4 + 16 · 5 + . . . (n terms),
(c) 2 · 4 − 3 · 5 + 4 · 6 − 5 · 7 + . . . (n terms).
MATH1034 1st Semester Algebra Lecture Manual 2017 59

5.2 Factorials
Definition.

For n ∈ N we define n! = 1 · 2 · 3 · · · (n − 2)(n − 1)n


0! = 1
n! is called the factorial of n and is pronounced “n factorial”.

Note. We can define n! inductively by 0! = 1 and n! = n(n − 1)!.


13! 8!
Worked Example 5.2.1. Find (a) 7!, (b) , (c) .
10! 11!
Solution. (a) 7! = 7 · 6 · 5 · 4 · 3 · 2 · 1 = 5040,
13! 13 · 12 · 11 · 10!
(b) = = 13 · 12 · 11 = 1716,
10! 10!
8! 8! 1
(c) = = .
11! 11 · 10 · 9 · 8! 990
(n + 1)!
Worked Example 5.2.2. Simplify .
(n − 1)!
Solution.
(n + 1)! (n + 1)n(n − 1)!
= = (n + 1)n.
(n − 1)! (n − 1)!

Definition.
Ç å
n n!
For n, k ∈ Z with 0 ≤ k ≤ n we define = .
k k!(n − k)!
Ç å
n
is pronounced “n choose k”.
k
Ç å Ç å Ç å Ç å
7 7 5 5
Worked Example 5.2.3. Calculate (a) , (b) , (c) , (d) .
3 4 1 0
Solution.
Ç å
7 7! 7 · 6 · 5 · 4!
(a) = = = 35,
3 3! (7 − 3)! 3 · 2 · 1 · 4!
Ç å
7 7!
(b) = = 35,
4 4! (7 − 4)!
Ç å
5 5!
(c) = = 5,
1 1! 4!
Ç å
5 5!
(d) = = 1.
0 0! 5!

n!
Note. Since = n(n − 1) · · · (n − k + 1), we have
(n − k)!

n(n − 1) · · · (n − k + 1)
Ç å
n
=
k 1 · 2···k

with k factors in both the numerator and denominator.


60 1st Semester Algebra Lecture Manual 2017 MATH1034

Theorem 5.4. For n, k ∈ Z with 0 ≤ k ≤ n we have:


Ç å Ç å
n n
1. = = 1,
0 n
Ç å Ç å
n n
2. = = n,
1 n−1
Ç å Ç å
n n
3. = ,
k n−k
n+1
Ç å Ç å Ç å
n n
4. + = if additionally k ≥ 1.
k k−1 k

Proof. 1., 2. Tutorial exercises.


Ç å Ç å
n n! n! n
3. = = = .
n−k (n − k)! (n − (n − k))! (n − k)! k! k
4.
Ç å Ç å
n n n! n!
+ = +
k k−1 k!(n − k)! (k − 1)!(n − (k − 1))!
n−k+1
ï ò
k
= n! +
k!(n − k + 1)! k!(n − k + 1)!
n+1
= n!
k!(n − k + 1)!
(n + 1)!
=
k!(n + 1 − k)!
n+1
Ç å
= .
k
Ç å Ç å
13 13
Worked Example 5.2.4. Calculate + .
9 10
Solution. Ç å Ç å Ç å
13 13 14 11 · 12 · 13 · 14
+ = = = 1001
9 10 10 1·2·3·4
Ç å
n
Note. 1. gives the number of ways of selecting k objects from n objects, when the order of selection is
k
irrelevant. Ç å
5 5!
For example, the number of ways of selecting 2 people from a group of 5 people is = = 10.
2 2!3!
You can confirm this by labelling 5 people A, B, C, D, E. Then the following selections of two people are possible:
AB, AC, AD, AE, BC, BD, BE, CD, CE, DE, i. e., there are 10 possible selections (remember order is irrelevant).
2. n! is only defined for non-negative integers. In other courses you will encounter the gamma function, Γ, which
is defined for all real numbers which are not non-positive integers and which satisfies Γ(n) = (n − 1)! for all n ∈ N.
3. Using the identity
n(n − 1) · · · (n − k + 1)
Ç å
n
=
k 1 · 2···k
we can define
α α(α − 1) · · · (α − k + 1)
Ç å
=
k k!
for all α ∈ R.
You do not need to remember this definition right now, but you will encounter it later this year.
MATH1034 1st Semester Algebra Lecture Manual 2017 61

Ç å Ç å
9 70
Tutorial 5.2.1. 1. Evaluate (a) , (b) .
3 64
2. Simplify the following expressions:
Ç å
n
k+1 10! 10! n!
(a) Ç å , (b) , (c) , (d) .
n (5!) 2 5! 2 (n + 1)! + (n − 1)!
k
X Q
3. Just as was used to denote summation, we use the symbol to denote products, i. e.,

n
Y
ar = a1 a2 a3 . . . an .
r=1

Show that
Ç å n n
n! 2n Y Y
(a) n = (2r − 1), (b) 2 n! =
n
(2r).
2 n r=1 r=1

n+2
Ç å Ç å
21 n
5. Find all possible values of nonnegative integers n such that = .
5 5 4

5.3 The Binomial Theorem


The binomial theorem gives an expansion of (a + b)n for any positive integer n and a, b ∈ R.

Let us consider expansions of (a + b)n for n = 0, 1, 2, 3, 4.

n = 0: (a + b)0 = 1
n = 1: (a + b) =
1
a + b
n = 2: (a + b)2 = a2 + 2ab + b2
n = 3: (a + b)3 = a3 + 3a2 b + 3ab2 + b3
n = 4: (a + b)4 = a4 + 4a3 b + 6a2 b2 + 4ab3 + b4

Table 1

We realise a pattern that emerges from these expansions:

1. (a + b)n yields an expansion with n + 1 terms.

2. Each term in the expansion has the form (coefficient · an−k · bk ), where k runs through all integers from 0 to n.
3. The coefficients are symmetrical about the vertical axis of the triangle in Table 1; in particular, the first and the
last coefficient equal 1, and the second and the penultimate coefficient equal n.

This triangular pattern is named Pascal’s triangle after the French Mathematician Blaise Pascal who first con-
structed it.
62 1st Semester Algebra Lecture Manual 2017 MATH1034

Pascal’s triangle
row 0: 1
row 1: 1 1
row 2: 1 2 1
row 3: 1 3 3 1
row 4: 1 4 6 4 1

It is easy to see that we have a pattern, up to row 4. The first and the last element in each row is 1, whereas each
of the elements in between is the sum of the two
Ç å elements above it, immediately to the left and the right. But from
n
Theorem 5.4, 4., we know that the numbers satisfy the same pattern, if n denote the row and k the position
k
in that row.

Hence, we can write Table 1 in the form (check that the entries in tables 1 and 2 coincide)
Ç å
0
row 0: (a + b) =
0
0
Ç å Ç å
1 1
row 1: (a + b) =
1
a + b
0 1
Ç å Ç å Ç å
2 2 2 2 2
row 2: (a + b) =
2
a + ab + b
0 1 2
Ç å Ç å Ç å Ç å
3 3 3 2 3 3 3
row 3: (a + b) =
3
a + a b + ab2
+ b
0 1 2 3
Ç å Ç å Ç å Ç å Ç å
4 4 4 3 4 2 2 4 4 4
row 4: (a + b) =
4
a + a b + a b + ab3
+ b
0 1 2 3 4

Table 2

This pattern is indeed true, and we now state and prove this theorem, using sigma notation:

Theorem 5.5 (Binomial Theorem). For any nonnegative integer and any real numbers a and b,

n Ç å
X n n−k k
(a + b)n = a b.
k=0
k

Proof. We prove the theorem by induction. Let P(n) be the statement of the theorem for the nonnegative integer n.
0 Ç å
X 0 0−k k
Then P(0) is the statement (a + b) =
0
a b.
k=0
k
LHS: (a + b)0 = 1;
0 Ç å Ç å
X 0 0−k k 0 0 0
RHS: a b = a b = 1.
k=0
k 0
So LHS equals RHS, and thus P(0) is true.
Now we assume that P(n) is true (induction hypothesis), and we want to show P(n + 1):

n+1 Ç
n + 1 (n+1)−k k
å
X
(a + b) n+1
= a b. (1)
k=0
k
MATH1034 1st Semester Algebra Lecture Manual 2017 63

We start with the LHS of (1):


(a + b)n+1 = (a + b)(a + b)n
n Ç å
X n n−k k
= (a + b) a b (by induction hypothesis)
k=0
k
n Ç å n Ç å
X n n+1−k k X n n−k k+1
= a b + a b (multiplying out). (2)
k=0
k k=0
k

Observing that we want to show that RHS of (2) equals RHS of (1), we aim at powers an+1 bk on the RHS of (2).
This is the case for the first sum on RHS of (2), but not for the second sum. But we see that an index shift does
the trick.
n Ç å n+1 Ç å
X n n+1−k k X n
(a + b) =
n+1
a b + an−(k−1) bk index shift
k=0
k k=1
k−1
Ç å n ñÇ å Ç åô Ç å
n n+1 0 X n n n
= a b + + a b +
n+1−k k
an−((n+1)−1) bn+1 (collect like powers).
0 k=1
k k − 1 (n + 1) − 1
(3)
Making use of Theorem 5.4, 1. and 4., i. e.,
n+1 n+1 n+1
Ç å Ç å Ç å Ç å Ç å Ç å Ç å
n n n n
=1= , =1= and + = ,
0 0 n n+1 k k−1 k
(3) becomes
n Ç
n + 1 n+1 0 X n + 1 n+1 k n + 1 0 n+1
Ç å å Ç å
(a + b) n+1
= a b + a b + a b
0 k=1
k n+1
n+1 Ç
n + 1 n+1 k
å
X
= a b
k=0
k

This shows that P(n + 1) is true.


By the principe of mathematical induction, P(n) is therefore true for all nonnegative integers.
Note. 1. The formula in Theorem 5.5 is clearly symmetrical in a and b. Therefore
n Ç å n Ç å
X n n−k k X n k n−k
(a + b) =
n
a b and (a + b) =
n
ab .
k=0
k k=0
k

Each of these formulae is called the Binomial Expansion of (a + b)n .


Ç å
n
2. The numbers are also called binomial coefficients.
k

Worked Example 5.3.1. Use the binomial theorem to expand (x + y)7 .


Solution. Ç å Ç å Ç å Ç å
7 5 2 7 4 3 7 3 4 7 2 5
(x + y) = x + 7x y +
7 7 6
x y + x y + x y + x y + 7xy6 + y7 .
2 3 4 5

Worked Example 5.3.2. Expand (a + 2b)5 .


Solution.
Ç å Ç å
5 3 5 2
(a + 2b) = a + 5a (2b) +
5 5 4
a (2b) +
2
a (2b)3 + 5a(2b)4 + (2b)5
2 3
= a5 + 10a4 b + 40a3 b + 80a2 b3 + 80ab4 + 32b5 .
64 1st Semester Algebra Lecture Manual 2017 MATH1034

Å ã6
1
Worked Example 5.3.3. Expand + 2x .
x
Solution.
Å ã6 Ç å Ç å Ç å
1 1 1 6 1 6 1 6 1 6
+ 2x = 6 + (6) 5 (2x) + (2x) +
2
(2x) +
3
(2x)4 + (2x)5 + (2x)6
x x x 2 x4 3 x3 4 x2 x
= x−6 + 12x−4 + 60x−2 + 160 + 240x2 + 192x4 + 64x6 .

Worked Example 5.3.4. Find the coefficient of x3 in the expansion of (2x − 3)8 .
Ç å
8
Solution.From the binomial theorem we know that this coefficient is the coeeficient of (2x)3 (−3)8−3 , which
3
is
8·7·6 3
(2) (−3)5 = −56(8)35 .
1·2·3
Å ã5
2 3
Worked Example 5.3.5. Find the constant term in the expansion of − x .
x2
Solution. The binomial expansion is
Å ã5 5 Ç å
2 X 5
− x3 = (2x−2 )5−k (−x3 )k ,
x2 k=0
k

and the term with summation index k has x power (−2)(5 − k) + 3k = 5k − 10. Since a constant term occurs if and
only if this power is 0, a constant term is obtained if and only if k = 2, and the corresponding coefficient is
Ç å
5 5−2
2 (−1)2 = 80.
2
n Ç å
X n k
Worked Example 5.3.6. Show that (1 + x) = x. n

k=0
k
Solution.By the binomal theorem,
n Ç å n Ç å
X n X n k
(1 + x) = n
(1) x =
n−k k
x.
k=0
k k=0
k

n Ç å
X n
Worked Example 5.3.7. Show that = 2n .
k=0
k
Solution. Putting x = 1 in Example 5.3.6, we get
n Ç å
X n
= (1 + 1)n = 2n .
k=0
k

Worked Example 5.3.8. Find the coefficient of x5 in the expansion of (1 + x)(2 + 3x)8 .
Solution.Writing
(1 + x)(2 + 3x)8 = (2 + 3x)8 + x(2 + 3x)8
we see that the coefficient of x5 in (1 + x)(2 + 3x)8 is the sum of the coeffients of x5 and x4 in (2 + 3x)8 .
But the coefficient of x5 in (2 + 3x)8 is
Ç å
8 8−5 5 8 · 7 · 6
2 3 = (8)35 = 56(8)35 .
5 1·2·3
MATH1034 1st Semester Algebra Lecture Manual 2017 65

And the coefficient of x4 in (2 + 3x)8 is Ç å


8 8−4 4
2 3 = 56(16)34 .
4
Therefore the coefficient of x5 in the expansion of (1 + x)(2 + 3x)8 is

56(8)35 + 56(16)34 = 56(8)34 (3 + 2) = 280(8)34 .

Tutorial 5.3.1. 1. Use the binomial theorem to expand the following expressions:
y 4 7 5
 Å ã
(a) (x + y)6 , (b) (2x − 6)4 , (c) x + , (d) 3x − 2 .
x x
Å ã7
3x
2. (a) Find the coefficient of x7 y−5 in the expansion of 2x − .
y
(b) Find the coefficient of x14 in the expansion of (2x − x3 )8 .
(c) Find the coefficient of x6 in the expansion of (1 − 3x)2 (x + 2)8 .
3. Find the constant term, if any, in each expansion.
ã9
x y 12
Å ã Å 3
x 3y
(a) − , (b) x3 2 + 2 .
y x y x
n Ç å
X n
4. Show that (−1)k = 0.
k=0
k
Chapter 6

Conic Sections

LEARNING OUTCOMES:
On completion of this chapter you should (tick the checkbox when you have mastered the skill)

 1. know and understand the definitions of quadratic forms and conic sections,
 2. know and understand the canonical form of quadratic forms,
 3. know and understand the classification of quadratic forms,
 4. be able to identify and classify quadratic forms,
 5. be able to sketch curves given by quadratic forms,
 6. know when and how to apply change of axes,
 7. know how to solve the examinable tutorial problems and the worked out problems from these notes on
this chapter.

6.1 Quadratic Forms


In this chapter we consider some curves in the x-y plane given implicitly, i. e., which satisfy an equation of the
form f (x, y) = 0.
The simplest curves obtained in this way are those for which f is linear, i. e., a polynomial of first degree in x and
y, say, f (x, y) = ax + by + c with a , 0 or b , 0. Then the equation of the curve becomes ax + by + c = 0, which
a
is a straight line with slope − if b , 0 and a vertical line if b = 0.
b
The next simplest type is when f is quadratic, i. e., of total degree two. That is, quadratic forms are given by

Ax2 + Bxy + Cy2 + Dx + Ey + F, (A , 0 or B , 0 or C , 0).

The curves given by these quadratic forms, i. e.,

Ax2 + Bxy + Cy2 + Dx + Ey + F = 0 (6.1.1)

are called conic sections since they can be obtained geometrically by intersecting a plane with a double cone, see
Huntley/Love, p. 657. However, here we will not be concerned with this geometric interpretation. We also note
that in some cases, (6.1.1) may have no solutions, a single point as solution, a line as solution, or a pair of lines as
solution, e. g., x2 + y2 + 1 = 0, x2 + y2 = 0, y2 = 0, x2 − y2 = 0, respectively. Such quadratic forms are called
degenerate and we will not consider them, i. e., we will only be concerned with non-degenerate quadratic forms.
In section 6.2 we will see that by a roation of axes and/or moving the origin, i. e., geometrically by a suitable
choice of the Cartesian coordinate system, every non-degenerate conic section given by (6.1.1) can be reduced to

66
MATH1034 1st Semester Algebra Lecture Manual 2017 67

one of the three canonical forms:

x2 y2 x2 y2
y2 = 4ax, + = 1, − =1 (a, b > 0). (6.1.2)
a2 b2 a2 b2

Now we will briefly discuss the curves given by these canonical forms.

6.1.1 Parabola

The curve given by the equation y2 = 4ax, a > 0, is clearly a parabola:

Figure 6.1: Parabola y2 = 4ax

6.1.2 Ellipse

x2 y2
The curve given by the equation + = 1, a, b > 0, has the following properties:
a2 b2

y2 x2
• Since 2
≥ 0, it follows that 2 ≤ 1 and hence that −a ≤ x ≤ a. Similarly, −b ≤ y ≤ b.
b a

x2 y2
• Since + is even in both x and y, the curve symmetrical about both axes.
a2 b2

2x 2y dy dy b2 x
• Implicit differentiation gives + = 0, and therefore = − if y , 0. In particular, the curve
a2 b2 dx dx a2 y
has vertical tangents at (±a, 0) and horizontal tangents at (0, ±b).

dy
• From the sign of we see that the curve is decreasing in the first and third quadrants, while it is increasing
dx
in the second and fourth quadrants.

The curve is an ellipse. If a = b, then the ellipse is a circle with centre at the origin and radius a.
68 1st Semester Algebra Lecture Manual 2017 MATH1034

x 2 y2
Figure 6.2: Ellipse + =1
a2 b2

6.1.3 Hyperbola
x2 y2
The curve given by the equation − = 1, a, b > 0, has the following properties:
a2 b2

y2 x2
• Since ≥ 0, it follows that ≥ 1 and hence that |x| ≥ a.
b2 a2

x2 y2
• Since − is even in both x and y, the curve symmetrical about both axes.
a2 b2

2x 2y dy dy b2 x
• Implicit differentiation gives − = 0, and therefore = if y , 0. In particular, the curve has
a2 b2 dx dx a2 y
vertical tangents at (±a, 0) and no horizontal tangents since x , 0 for all points (x, y) on the curve.

dy
• From the sign of we see that the curve is increasing in the first and third quadrants, while it is decreasing
dx
in the second and fourth quadrants.

y2 b2 b2 y b b
• From = − it follows that lim = ± . Hence the lines y = ± x are oblique asymptotes.
x 2 a2 x 2 x→±∞ x a a

The curve is a hyperbola.

x2 y2
Figure 6.3: hyperbola − =1
a2 b2
MATH1034 1st Semester Algebra Lecture Manual 2017 69

Note. 1. Since we will show in the next section that every non-degenerate quadratic form can be reduced to one of
the three canonical forms, it follows that each non-degenerate quadratic form represents one of the three curves:
parabola, ellipse, hyperbola.
2. The canonical form for a parabola has one quadratic and one linear term, whereas the canonical forms for the
ellipse and the hyperbola have two quadratic and one constant term, with the quadratic terms for the ellipse having
the same sign, whereas they have opposite signs for the hyperbola.
3. The ellipse cuts both axes twice, whereas the hyperbola only cuts one axis. The asymptotes of the hyperbola
can be found by replacing the 1 on the right hand side by 0.
4. Other canonical forms for the hyperbola and the parabola are obtained when x and y are interchanged and when
a is allowed to be negative for parabolas, see the examples below.

Worked Example 6.1.1. Identify the following curves and sketch roughly:
(a) y = 2x2 , (b) y2 = 2x, (c) x = −2y2 , (d) 2y = −x2 ,
(e) 2x2 + y2 = 2, (f) 2x2 − y2 = 2, (g) 2y2 − x2 = 2.

Solution. The quadratic forms (a)–(d) are parabolas, with the linear term indicating the axis, along the positive
axis if the quadratic and the linear term (on different sides of the equation) have the same sign, and along the
negative axis if the quadratic and the linear term (on different sides of the equation) have opposite signs. Hence
the sketch, in the same diagram:

y y = 2x2
4

3 y2 = 2x

2
x = −2y2

x
−4 −3 −2 −1 1 2 3 4
−1

−2

−3

−4
2y = −x2

The quadratic form (e) is an ellipse:


70 1st Semester Algebra Lecture Manual 2017 MATH1034

y
1 2x2 + y2 = 2

x
−1 1
−1

The quadratic forms (f) and (g) are hyperbolas:

y 2x2 − y2 = 2
4
3
2 2y2 − x2 = 2
1
x
−5 −4 −3 −2 −1 1 2 3 4 5
−1
−2
−3
−4

Tutorial 6.1.1. 1. Sketch the graphs of the curves given by the equations below. Include the asymptotes when
sketching hyperbolas.
x2 y2 x2 y2 x2 y2 x2 y2
(a) + = 1, (b) + = 1, (c) y2 = −4x, (d)− = 1, (e) − = 1.
4 9 9 4 4 9 9 4
x2 y2
2. Show that a point (x, y) in the Euclidean plane lies on the ellipse 2 + 2 = 1 if and only if there is an angle θ
a b
such that x = a cos θ and y = b sin θ.
3. Suppose a ladder stands on horizontal ground, leans against a vertical wall and gradually slips downward,
keeping in contact with floor and wall at all times. Show that any point of the ladder other than an endpoint
describes an arc of an ellipse. (Hint: express the co-ordinates of the point in terms of θ, the angle between the
ladder and the ground.)

6.2 Change of Axes


Here we will show that the general non-degenerate quadratic form can be reduced to canonical form if we choose
a suitable co-ordinate system, without changing the geometry of the conic section. Admissible co-ordinate trans-
formations are translations (shifts) and rotations, which leave angles and distances unchanged.

6.2.1 Translation of axes

If B = 0 in (6.1.1) and the quadratic form is non-degenerate, then this equation can be transformed, by a translation
of axes, into canonical form as in Note 4 of the previous section.
Case I: A = 0 or C = 0.
We assume now A = 0, the case C = 0 is similar. Then C , 0 since the total degree of the quadratic form is two.
MATH1034 1st Semester Algebra Lecture Manual 2017 71

We complete the square in the quadratic form:


Å ã
E
Cy2 + Dx + Ey + F = C y2 + y + Dx + F
C
E 2 E2
Å ã
=C y+ + Dx + F − .
2C 4C 2
If D would be zero, then the solution to
E 2 E2
Å ã
C y+ + Dx + F − =0
2C 4C 2
would either be a horizontal line, two horizontal lines or the empty set, i. e., we would have a degenerate quadratic
form. Therefore we have D , 0. Putting now
E F E2 D
k=− , h=− + and a = − ,
2C D 4DC 2 4C
u = x − h, v = y − k,
we have
Cy2 + Dx + Ey + F = C (y − k)2 + D(x − h)
= C v2 − 4au .


Therefore, in the u-v co-ordinate system, the equation Cy2 + Dx + Ey + F = 0 becomes


v2 = 4au,
which is of the canonical form as mentioned in Note 4 of the previous section. It will be sufficient, in general, to
leave the solution in this form.
If a > 0, we have obtained the canonical form of a parabola as in (6.1.2).
If we also want the equation in the form (6.1.2) in case a < 0, we replace u with −u and v with −v, which
corresponds to a rotation of the coordinate system through the angle π, see the next subsection.
If C = 0, the above procedure would give u2 = av, with suitable values of h and k. Replacing v with u and −u
π
with v, which corresponds to a rotation of the co-ordinate axes through the angle , the above equation becomes
2
v2 = au, and a further rotation by π may be needed to achieve the canonical form (6.1.2) of the parabola.
The geometric meaning of the shift is sketched below:

y v

u
(h, k)

x
(0, 0)

Case II: A , 0 and C , 0, and A and C have the same sign.


We complete the squares:
Å ã Å ã
D E
Ax2 + Cy2 + Dx + Ey + F = A x2 + x + C y2 + y + F
A C
ã2
E 2 D2 E2
Å Å ã
D
= A x+ +C y+ +F− 2 − .
2A 2C 4A 4C 2
72 1st Semester Algebra Lecture Manual 2017 MATH1034

This quadratic form is non-degenerate if and only if


D2 E2
G= 2
+ −F ,0 and has the same sign as A and C.
4A 4C 2
Putting now
… …
E D G G
k=− , h=− , a= and b= ,
2C 2A A C
u = x − h, v = y − k,

we have
(x − h)2 (y − k)2
ï ò
Ax2 + Cy2 + Dx + Ey + F = G + −1
a2 b2
v2
ï 2 ò
u
=G 2 + 2 −1 .
a b
Therefore, in the u-v co-ordinate system, the equation Ax2 + Cy2 + Dx + Ey + F = 0 becomes
u2 v2
+ = 1,
a2 b2
which is the canonical form (6.1.2) of the ellipse.
Case III: A , 0 and C , 0, and A and C have opposite signs.
We complete the squares:
Å
ã Å ã
D E
Ax + Cy + Dx + Ey + F = A x + x + C y + y + Dx + F
2 2 2 2
A C
ã2
E 2 D2 E2
Å Å ã
D
= A x+ +C y+ +F− 2 − .
2A 2C 4A 4C 2
This quadratic form is non-degenerate if and only if
D2 E2
G= + − F , 0.
4A2 4C 2
If G has the same sign as A, we put
… …
E D G G
k=− , h=− , a= and b = − ,
2C 2A A C
u = x − h, v = y − k,

and we get
(x − h)2 (y − k)2
ï ò
Cy2 + Dx + Ey + F = G − − 1
a2 b2
v
ï 2 2 ò
u
=G 2 − 2 −1 .
a b
Therefore, in the u-v co-ordinate system, the equation Ax2 + Cy2 + Dx + Ey + F = 0 becomes
u2 v2
− = 1,
a2 b2
which is the canonical form (6.1.2) of the hyperbola.
Similarly, if A and G have opposite signs, then we obtain the equation
v2 u2
− = 1,
a2 b2
MATH1034 1st Semester Algebra Lecture Manual 2017 73

π
which can be transformed to the canonical form (6.1.2) by a rotation through the angle .
2
Note. 1. You should not remember the details of the formulas but rather the principle of completing the squares
and how to classify the quadratic form. In the examples below you will see how to do this reduction to canonical
form in concrete cases.
2. You will see if a quadratic form is non-degenerate along the way while you reduce it to canonical form. There
is no need to investigate non-degeneracy separately.

Worked Example 6.2.1. Write the following equations in canonical form, and identify and sketch the curve
(showing old and new axes):
(a) y = −3x2 + 12x − 13, (b) x2 + 2x + 2y2 − 8y + 7 = 0,
(c) −2x2 + 4x + y2 = 3, (d) x2 − 2x + y2 = 0.

Solution. (a) Since we only have one quadratic term, this quadratic form, if non-degenerate, gives a parabola.
Completing the square we have

3x2 − 12x + y + 13 = 3(x2 − 4x) + y + 13


ï ò
1
= 3 (x − 2) + (y + 13) − 4
2
3
ï ò
1
= 3 (x − 2)2 + (y + 1) ,
3

and putting
h = 2, k = −1, u = x − h = x − 2, v = y − k = y + 1,
the canonical form of y = −3x2 + 12x − 13 is
1
u2 = − v.
3
This gives the sketch

y v

x
(0, 0)
(2, −1)
u

y = −3x2 + 12x − 13

(b) Here we have two quadratic terms with the same sign. So this quadratic form, if non-degenerate, gives an
74 1st Semester Algebra Lecture Manual 2017 MATH1034

ellipse (the coefficients of x2 and y2 are different, so it is not a circle). Completing the squares we have

x2 + 2x + 2y2 − 8y + 7 = (x + 1)2 + 2(y − 2)2 + 7 − 1 − 8


= (x + 1)2 + 2(y − 2)2 − 2,

and putting
h = −1, k = 2, u = x − h = x + 1, v = y − k = y − 2,
the canonical form of x2 + 2x + 2y2 − 8y + 7 = 0 is

u2
+ v2 = 1.
2
This gives the sketch

v y

x2 + 2x + 2y2 − 8y + 7 = 0
(−1, 2)
u

x
(0, 0)

(c) Here we have two quadratic terms with opposite signs. So this quadratic form, if non-degenerate, gives a
hyperbola. Completing the squares we have

−2x2 + 4x + y2 − 3 = −2(x − 1)2 + y2 − 3 + 2


= −2(x − 1)2 + y2 − 1,

and putting
h = 1, k = 0, u = x − h = x − 1, v = y − k = y,
the canonical form of −2x2 + 4x + y2 = 3 is
−2u2 + v2 = 1.
This gives the sketch

y v
−2x2 + 4x + y2 = 3

(1, 0)
x, u
(0, 0)
MATH1034 1st Semester Algebra Lecture Manual 2017 75

(d) Here we have two quadratic terms with the same coefficient. So this quadratic form, if non-degenerate, gives
a circle. Completing the squares we have

x2 − 2x + y2 = (x − 1)2 + y2 − 1,

and putting
h = 1, k = 0, u = x − h = x − 1, v = y − k = y,
the canonical form of x − 2x + y = 0 is
2 2

u2 + v2 = 1.
This gives the sketch

x2 − 2x + y2 = 0
y v

(1, 0)
x, u
(0, 0)

6.2.2 Rotation of axes


If B , 0 in (6.1.1), then this cross term can be removed by a suitable rotation of axes. If necessary, this may be
followed by a translation of axes, as dealt with above, to transform the quadratic form into its canonical form.
We now explain the method of rotation of the axes. Consider the following sketch:

y
v
P

θ
θ−α
α
x
(0, 0)

The old Euclidean x-y co-ordinate system is rotated through an angle α to obtain a new Euclidean u-v co-ordinate
system. Consider a point P which has polar co-ordinates (r, θ) with respect to the old x-y co-ordinate system. Then
P has polar co-ordinates (r, θ − α) in the new co-ordinate system. Thus we have

x = r cos θ and y = r sin θ (old axes),


u = r cos(θ − α) and v = r sin(θ − α) (new axes).

Therefore

u = r cos θ cos α + r sin θ sin α,


v = −r cos θ sin α + r sin θ cos α,
76 1st Semester Algebra Lecture Manual 2017 MATH1034

which gives

u = x cos α + y sin α,
v = −x sin α + y cos α.

Conversely, if the point P has polar coordinates (r, ψ) in the new co-ordinate system, it has polar co-ordinates
(r, ψ + α) in the old co-ordinate system, and therefore

x = u cos α − v sin α,
y = u sin α + v cos α.

A substitution of these equations into

Ax2 + Bxy + Cy2 + Dx + Ey + F

shows that the coefficient of the new cross term, i. e., the coefficient of uv, becomes

−2A cos α sin α + B(cos2 α − sin2 α) + 2C sin α cos α = (C − A) sin 2α + B cos 2α,

which equals zero if and only if


A−C
cot 2α = .
B
π
Since cot is π-periodic, the angle α is only unique up to adding multiples of , and for convenience we can choose
2
π
0<α< .
2
This allows us to find the angle α and thus sin α and cos α. However, if we do not want to sketch the curve, we do
not need to find the angle explicitly, and we can find cos α and sin α as follows:
cot 2α
cos 2α =
cosec 2α
cot 2α
= √
1 + cot2 2α
A−C |B|
= p .
B + (A − C) B
2 2

Therefore
1 √ 1 √
cos α = √ 1 + cos 2α, sin α = √ 1 − cos 2α
2 2
(note that both cos α and sin α are positive) give

1 A−C |B|
cos α = √ 1+ p ,
2 B2 + (A − C)2 B
1 A−C |B|
sin α = √ 1− p .
2 B2 + (A − C)2 B

After this rotation a translation may be needed to transform the quadratic form to canonical form.
Note. 1. The new axes are called principal axes of the quadratic form. They are of great importance in applica-
tions such as statistics, geometry, physics and engineering.
π
2. The principal axes are perpendicular to each other, and a rotation through interchanges the principal axes,
2
giving another canonical form of the equation.

Worked Example 6.2.2. Write the following equations in canonical form, and identify and sketch the curve
(showing old and new axes):
MATH1034 1st Semester Algebra Lecture Manual 2017 77


(a) xy = 2, (b) 11x2 + 4 3xy + 7y2 = 1.
π π
Solution. (a) Here we have A = C = 0, and therefore cot 2α = 0, which gives 2α = and hence α = . This
2 4
gives x and y in terms of the new co-ordinates as
1
x = u cos α − v sin α = √ (u − v),
2
1
y = u sin α + v cos α = √ (u + v).
2

Therefore
1 1 1
xy = √ (u − v) √ (u + v) = (u2 − v2 ),
2 2 2
and xy = 2 becomes
u2 − v2 = 4,
which is the canonical form of a hyperbola.
The sketch is as follows:

y
3
v u
2
1
−3 −2 −1 α x
1 2 3
−1
−2
−3


(b) Here we have A = 11, B = 4 3, and C = 7, and therefore
11 − 7 1
cot 2α = √ = √ ,
4 3 3
π π
which gives 2α = and hence α = . This gives x and y in terms of the new co-ordinates as
3 6
1 Ä√ ä
x = u cos α − v sin α = 3u − v ,
2
1Ä √ ä
y = u sin α + v cos α = u + 3v .
2

Therefore
√ 11 Ä √ ä2 √ Ä √ äÄ √ ä 7Ä √ ä2
11x2 + 4 3xy + 7y2 = 3u − v + 3 3u − v u + 3v + u + 3v
4 4
11 Ä 2 √ ä Ä 2 √ ä 7Ä 2 √ ä
= 3u + v − 2 3uv + 3u − 3v + 2 3uv +
2 2
u + 3v2 + 2 3uv
4 4
= 13u2 + 5v2 .

Hence 11x2 + 4 3xy + 7y2 = 1 becomes
13u2 + 5v2 = 1,
which is the canonical form of an ellipse.
The sketch is as follows:
78 1st Semester Algebra Lecture Manual 2017 MATH1034

y
v
0.4

u
0.2

α x
−0.4 −0.2 0.2 0.4
−0.2

−0.4

Tutorial 6.2.1. 1. Write the following equations in canonical form, and identify and sketch the curve (showing
old and new axes):
(a) 2x2 − 4xy − y2 + 8 = 0, (b) 5x2 + 4xy + 5y2 = 9, (c) 11x2 + 24xy + 4y2 − 15 = 0.
1. Write the following equation in canonical form, and identify and sketch the curve (showing old and new axes):
9x2 − 4xy + 6y2 − 10x − 20y = 5.
Appendix A

Some Basic Mathematical Notions

This chapter serves as a reference to basic mathematical notions. You may consult this section whenever you are
uncertain about mathematical notations or reasoning. You are advised to read through this chapter every now and
again to refresh your familiarity with basic notations and concepts.
You will not be examined directly on the contents of this chapter.

A.1 Basic Mathematical Terms, Sets and Forms of Reasoning


A.1.1 Basic Mathematical Terms
A statement is an expression which is either true or false. A statement may be concerned with a single object or
with a variety of objects.

Example A.1.1. Decide whether the following statements are true or false:
a) 2 + 4 = 6,
b) 42 = 29,
c) If it rains today, then it will rain tomorrow.
Solution. Statements a) and b) are concerned with a single object. Clearly, a) is true and b) is false.
c), on the other hand, is concerned with many objects. For any particular chosen day, it may or may not rain on
the following day, and there are definitely rainy days where the following day is without rain. Therefore we have
at least one instance where the statement is false. Hence statement c) is false.

Recall that a statement must be either true or false. Hence, if a statement P is true, then its denial “not P” (written
¬P or ∼ P) is false, and vice versa.
A definition describes an object or concept, possibly involving previously defined concepts or objects.
A theorem is a true statement which is derived by forms of reasoning from true statements and definitions. A
theorem of minor importance or which is a step towards one or more important results is often called a lemma or
a proposition.
This, however, presents us with a dilemma since every true statement would be based on another true statement,
which would have to be proved. Hence mathematics needs some statements which are assumed to be true:
An axiom is a statement that is assumed to be true.
Axioms are usually statements which appear to be self-evidently true. This is because most mathematical axioms
were chosen so that they are consistent with experience. For example, the statement

“Given a line and a point not on that line, one and only one line can be drawn through that point
parallel to the given line.”

seems like a reasonable axiom to assume true for planar geometry.

A1
A2 1st Semester Algebra Lecture Manual 2017 MATH1034

Axioms are the most elementary building blocks of the mathematical universe, and are normally accepted by
(almost) everyone in the mathematical community because they are consistent with mathematical and practical
experience. You will be presented with another important axiom in Subsection A.2.1.

A.1.2 Forms of Reasoning

An implication is a statement of the form “if p then q” or, equivalently, “p implies q”,
symbolically written as “p ⇒ q”, where both p and q are statements.
p is called the hypothesis and q is called the conclusion.

Example A.1.2. The statements below are all implications.


a) If it is raining, then there are clouds in the sky.
b) x = 2 implies that x2 = 4.
c) x2 = 4 implies that x = 2.
d) x + 3 = 5 ⇒ x = 2.
e) If triangles are congruent, then they are similar.

The converse of p ⇒ q is q ⇒ p.

Note that knowing the truth value of an implication does not determine the truth value of the converse of that
implication.

Example A.1.3. Referring to Example A.1.2 we have


The converse of statement a) is “If there are clouds in the sky, then it is raining.” It is a false statement.
The converse of statement b) is “x2 = 4 implies that x = 2.” It is a false statement.
The converse of statement c) is “x = 2 implies that x2 = 4.” It is a true statement.
The converse of statement d) is “x = 2 ⇒ x + 3 = 5.” It is a true statement.
The converse of statement e) is “If triangles are similar, then they are congruent.” It is a false statement.

If two statements p and q have the same truth value we say “p if and only if q” and we write p ⇔ q.
This is called an equivalence or a double implication.

Note. 1. If the statement depends on one or more variables x, then p(x) ⇔ q(x) means that the statements p(x)
and q(x) have the same truth value for all x which can be substituted into p(x) or q(x). For further clarification,
see Subsection A.1.4.
2. It is important to note that p ⇔ q if and only if both p ⇒ q and q ⇒ p are true.

Example A.1.4. a) Let p be the statement x = 3 and let q be the statement 4x = 12. Then p ⇒ q and q ⇒ p are
both true, and hence p ⇔ q is true.
b) Let p be the statement x = 3 and let q be the statement x2 = 9. Then p ⇒ q is true but q ⇒ p is false (because
when x = −3, p is false but q is true), and hence p ⇔ q is false.

Note. Instead of ¬(p ⇒ q) one mostly writes p ; q, and instead of ¬(p ⇔ q) one mostly writes p < q.

Every implication p ⇒ q has a contrapositive: ¬q ⇒ ¬p.


MATH1034 1st Semester Algebra Lecture Manual 2017 A3

The contrapositive of a statement is logically equivalent to the original statement, i. e., the contrapositive of a true
statement is true and the contrapositive of a false statement is false.

Example A.1.5. a) “If it is raining, then it is cloudy” is a true statement.


Its contrapositive is “If it is not cloudy, then it is not raining”, which is also a true statement.
b) “If x < y, then 2x > 2y” is a false statement. Therefore its contrapositive “If 2x ≤ 2y, then x ≥ y” is also false.
c) a = 3 ⇒ 2a = 6 has contrapositive 2a , 6 ⇒ a , 3.

Note the difference between the contrapositive of a statement and the converse of a statement. The con-
verse of a true statement may or may not be true, whereas the contrapositive is an equivalent statement
and may be used in place of the original.

If p and q are statements, then the statement

• “p and q”, in symbolic notation p ∧ q, is true if and only if both p and q are true,
• “p or q”, in symbolic notation p ∨ q, is true if and only if at least one of p and q are true.

Note. When speaking English, we often use “and” and “or” ambiguously. It is important to use these terms
unambiguously in mathematics.
We conclude this subsection with a truth table.

p q p⇒q p⇔q p∧q p∨q


T T T T T T
T F F F F T
F T T F F T
F F T T F F

Observe that p ⇒ q is true if p is false, no matter whether q is true or false. Indeed, since the hypothesis is not
satisfied, “if p then q” does not ask you to check q for its truth, in other words, p ⇒ q is a void statement, which
is true. In plain English, this means if you do not say anything, you tell the truth. This is consistent with the above
truth table. For example, if p is the void statement, then p ∧ q = q for any statement q, and the truth table is only
valid if the void statement is true.

A.1.3 Methods of proof


A statement or theorem may be proved as follows

• directly, by starting with something known (a definition, axiom or a statement which has been proved to be
true) and then using the laws of logic and other known results to proceed to the desired result or
• indirectly, by demonstrating that the denial of the statement or theorem cannot be true. This is called proof
by contradiction and is based on the logical equivalence of the contrapositive.

A conjecture is a statement which may or may not be true.


Note. A single counterexample is sufficient to disprove a conjecture, but a conjecture cannot be proved by verify-
ing it for a number of specific examples (that is, examples which do not cover all instances of the conjecture). To
prove that a conjecture or theorem is true, the general case must be shown to be logically valid.

Example A.1.6. a) To disprove the statement “ f (x) = x2 is one-to-one” we need only a single example whereby
we get the same value for f (x) for two different x-values. For example, x = 2 and x = −2 both yield f (x) = 4.
This constitutes a counterexample to the statement, so the statement is false.
A4 1st Semester Algebra Lecture Manual 2017 MATH1034

b) To prove the statement “g(x) = 3x + 1 is one-to-one” we must prove in general that no two different x-values
yield the same value of g(x).
Here we present a proof by contradiction.
Suppose the statement “g(x) = 3x + 1 is one-to-one” is false, i. e., suppose that g is not one-to-one. Then there
exist x1 , x2 such that g(x1 ) = g(x2 ), and we argue

g(x1 ) = g(x2 ) ⇔ 3x1 + 1 = 3x2 + 1


⇒ 3x1 = 3x2
⇒ x1 = x2 .

But this contradicts the initial assumption that x1 , x2 . So the statement “g is not one-to-one” is false, and hence
its negation “g is one-to-one” is true.

Note. In the above proof, “⇒” can be replaced with “⇔”, and “⇔” can be replaced with “⇒”. However, when
substituting a definition, it is often didactically advantageous to write “⇔” even when the proof only requires
“⇒”.

A.1.4 Sets
A set is a well-defined collection of objects. These objects are then called the elements of the set. Sets can be
written by listing all its elements, inside curly brackets and separated by commas. For example,

C = {Africa, Antarctica, Australia, Asia, Europe, North America, South America}

is the set of all continents. Note that the order of the listing is immaterial. For example,

A = {1, 2, 3, 4} and B = {1, 3, 2, 4}

are the same set, and we write


A = B.
Listing all elements in this way is only feasible for sets with few elements, and for larger sets we can use the
“dots” or “ellipsis” notation. For example,

Letters = {a,b,c,. . . ,z},

where the rule for the general element of the set should be clear from listing a few elements of the set. If the set
has infinitely many elements, then there is no last element to be listed. For example,

W = {a,aa,aaa,aaaa, . . . }.

The most general way to describe a set is to use set builder notation:

S = {x | A(x)} or S = {x : A(x)},

where A(x) is a statement which involves the quantity x, and S is the set consisting of all x for which A(x) is true.
If A is a set, then instead of “x is an element of A” you can also say “x belongs to A” and the symbolic notation

x∈A

is used for this statement.


A set A is called a subset of B, in symbolic notation A ⊂ B, if every element of the set A also belongs to the set B.
The intersection of two sets A and B, denoted by A ∩ B, is the set of all elements that belong to both A and B:

A ∩ B = {x | x ∈ A and x ∈ B}.
MATH1034 1st Semester Algebra Lecture Manual 2017 A5

The union of two sets A and B, denoted by A ∪ B, is the set whose elements belong to A or B:
A ∪ B = {x | x ∈ A or x ∈ B}.

When dealing with the set-builder notation, the statement A(x) often applies to a “universal set” X. Then instead
of
A = {x | x belongs to X and A(x)}
one mostly writes
A = {x ∈ X | A(x)}.
For example, if N is the set of natural numbers, see Section A.2.1, then we would write the set of all natural
numbers strictly between 18 and 956 by
{x ∈ N | 18 < x < 956}
instead of
{x | x ∈ N and 18 < x < 956}.
Finally, there is one particular set which deserves mentioning: the set which does not contain any element, called
the empty set, which is denoted by ∅.

A.2 Real Numbers and Intervals


A.2.1 Definition and Properties of Real Numbers
In this subsection the construction of real numbers and their fundamental properties are presented without proof.
The natural numbers are all numbers 1, 2, 3, . . . . The set of natural numbers is denoted by N. In set notation
this can be written as
N = {1, 2, 3, . . . }.
Note. 1. The existence of the natural numbers is an axiom, called Peano’s axiom, named after the Italian mathe-
matician Giuseppe Peano (1858-1932). Mathematical induction, see Chapter 4, is based on Peano’s axiom.
2. There is no general consensus if the natural numbers should start with 0 or 1, and the purpose of their use often
determines the particular choice. Hence, when you come across the notion of natural numbers, make sure that you
know how the author(s) of that publication have defined natural numbers. Also note that subscripts or superscripts
may be added to the symbol N to denote distinct versions of the set of natural numbers, such as
N∗ = {1, 2, 3, . . . } and N0 = {0, 1, 2, 3, . . . }.

The operations of addition and multiplication are defined on integers. However neither of their inverses is defined
in general. For addition, this means that for general a, b ∈ N the equation
a+x=b
may not have a solution in N. For example, there is no natural number x which satisfies 9 + x = 4. Enlarging the
set of natural numbers by all solutions to equations of the form a + x = b we arrive at the integers:
The integers consist of all natural numbers, negatives of natural numbers, and zero. The set of integers is denoted
by Z. In set notation this can be written as
Z = {. . . , −3, −2, −1, 0, 1, 2, 3, . . . }.

Now, for all a, b ∈ Z the equation a + x = b has a (unique) solution x ∈ Z. This is in particular true when a, b ∈ N.
However, multiplication in Z is not invertible in general. For example, there is no integer x satisfying 3x = 2.
Note that 0 · a = 0 for all a ∈ Z, so that 0 · x = b does not have a (unique) solution, if any. However, for integers
p
p, q with q , 0 we introduce the number as the formal solution to the equation
q
qx = p
A6 1st Semester Algebra Lecture Manual 2017 MATH1034

and call it a rational number. Note that the representation of rational numbers is never unique. Indeed, if p1 , q1 ,
p2 , q2 are integers with q1 , 0 and q2 , 0, then
p1 p2
= ⇔ p1 q2 = p2 q1 .
q1 q2

The set of rational numbers is denoted by Q. In set notation this can be written as
ß ™
p
Q= : p, q ∈ Z, q , 0 .
q

a
Note that any integer a can be written as so that every integer is also a rational number.
1
Here
√ our2 construction ends. But our journey into numbers is not yet finished since for example the positive solution
2 of x = 2, which represents the length of the diagonal of a square with side length 1, and the number π, which
represents the circumference of a circle with diameter 1, are mathematical objects which are not rational numbers.
The real numbers are obtained from the rational numbers by filling in all the “holes” on the number line. The set
of real numbers is denoted by R.
The existence of the real number depends on another axiom, the Archimedean axiom, or, equivalently, the Com-
pleteness Axiom. In Calculus you will learn, without proof, that every increasing bounded sequence has a limit.
This statement is equivalent to the Completeness Axiom. The formulation and consequences of the Archimedean
axiom will be presented in second year in Basic Analysis.
The irrational numbers consist of all real numbers which are not rational numbers. There is no particular symbol
for the irrational numbers. However, one can use the symbols R \ Q or, more briefly Q0 , where, in general, A \ B,
called the complement of B in A, is the set of all elements in A which do not belong to the set B.
We can add and multiply real numbers, and you should recall the laws for these operations, in particular the
associative laws and the distributive law. You should also recall subtraction and division as inverses of addition
and multiplication, respectively. You can find these rules and constructions in Huntley/Love, Section 1.1, pp.
13–16 and in Thomas, Section A.4, p. AP-9.
For convenience, we list these laws, as stated in Thomas:

A1 a + (b + c) = (a + b) + c for all a, b, c ∈ R.

A2 a + b = b + a for all a, b ∈ R.

A3 There is a real number called “0” such that a + 0 = a for all a ∈ R.

A4 For each a ∈ R there is b ∈ R such that a + b = 0.

M1 a(bc) = (ab)c for all a, b, c ∈ R.

M2 ab = ba for all a, b ∈ R.

M3 There is a real number called “1” such that a · 1 = a for all a ∈ R.

M4 For each a ∈ R with a , 0 there is b ∈ R such that ab = 1.

D a(b + c) = ab + ac for all a, b, c ∈ R.

[Note the typesetting error in the distributive law D in Thomas.]


1
The (unique) number b in A4 is denoted by −a, and the (unique) number b in M4 is denoted by or, equivalently,
a
a−1 .
Real numbers can be ordered: If a and b are two real numbers, then we say that “b is greater than a” or “b is larger
than a” or “a is less than b” or “a is smaller than b” if b is to the right of a on the real number line, and we write

b>a or, equivalently a < b.


MATH1034 1st Semester Algebra Lecture Manual 2017 A7

It is convenient to introduce the following notation for two real numbers a and b:

a ≤ b ⇔ a < b or a = b,
a ≥ b ⇔ a > b or a = b.

From a theoretical point of view, ≤ is more useful than <, and hence the order properties are written in terms of ≤
in Thomas, Section A4, p. AP-9, where a, b and c are real numbers:

O1 For any a and b, a ≤ b or b ≤ a or both.

O2 If a ≤ b and b ≤ a, then a = b.
O3 If a ≤ b and b ≤ c, then a ≤ c.
O4 If a ≤ b, then a + c ≤ b + c.

O5 If a ≤ b and 0 ≤ c, then ac ≤ bc.

Other rules for inequalities can be derived from (O1)–(O5), as the following example shows.

Example A.2.1. If a < b and c < 0, then ac > bc.

Proof. Assume the statement is false., i. e., ac ≤ bc for some a < b and c < 0. Then

0 = c + (−c) ≤ 0 + (−c) = −c (by O4 since c ≤ 0).

Now
a ≤ b and 0 ≤ −c ⇒ a(−c) ≤ b(−c) (by O5).
But
0 = a(c + (−c)) = ac + a(−c) ⇒ a(−c) = −ac.
Similarly, b(−c) = −bc. Hence we have shown

−ac ≤ −bc.

Adding ac + bc on both sides and observing O4, A1 and A2 we get

bc ≤ ac.

Since we assumed ac ≤ bc, it follows that ac = bc by (O2). Since c > 0 we have c , 0, and c−1 exists by M4.
Then M1 and M3 give
a = a(cc−1 ) = (ac)c−1 = (bc)c−1 = b(cc−1 ) = b.
But this contradicts a < b, so that our assumption that the statement of this example is false has been shown to be
incorrect. Hence the statement is correct, i. e., we have ac > bc whenever a < b and c < 0.
A8 1st Semester Algebra Lecture Manual 2017 MATH1034

A.2.2 Intervals
Intervals are subsets of the real number line which correspond geometrically to line segments. If a and b are real
numbers with a < b, then we have the following types of intervals, see Thomas, Section B1, p. AP-26.

Notation Set description Type Picture


(a, b) {x ∈ R | a < x < b} Open a b
[a, b] {x ∈ R | a ≤ x ≤ b} Closed a b
[a, b) {x ∈ R | a ≤ x < b} Half-open a b
(a, b] {x ∈ R | a < x ≤ b} Half-open a b
(a, ∞) {x ∈ R | x > a} Open
a
[a, ∞) {x ∈ R | x ≥ a} Closed
a
(−∞, b) {x ∈ R | x < b} Open
b
(−∞, b] {x ∈ R | x ≤ b} Closed
b
(−∞, ∞) R Both open and closed

Here ∞ and −∞, called “infinity” and “minus infinity”, respectively, are mere convenient symbols and do not
represent real numbers.

A.3 The Power of Notation and Reasoning


Mathematics has a powerful language by which we can express ideas clearly and unambiguously. Just as poor
grammar when speaking or writing in any language leads to a lack of clarity and to misunderstandings, so incorrect
and careless notation and reasoning in mathematics often leads to ambiguities and confusion. So please take care
with your notation and reasoning.

You might also like